Anda di halaman 1dari 114

INS INSIGHTS IAS PRELIMS TEST SERIES - 2017

Prelims 2017 Test 24 Full Length

1. Many Indian cities have obtained credit ratings, which are


necessary for mobilization of resources for Smart City
Mission and AMRUT like schemes. Consider the following
in this reference.
1. Higher credit ratings help ensure better sale of
Municipal Bonds.
2. No Indian city has received Investment Grade ratings.
Which of the above is/are correct?
a) 1 only
b) 2 only
c) Both 1 and 2
d) None
Solution: a)
Justification: With the exercise of Credit Rating of cities and
towns gaining momentum, 94 of the 500 cities included in
Smart City Mission and Atal Mission for Rejuvenation and
Urban Transformation (AMRUT) have obtained such ratings
which are necessary for issuing Municipal Bonds for
mobilization of resources.
Statement 2: 55 of these cities have got Investment Grade
ratings. So many cities getting Investment Grade rating was
better than what was thought of about the financial situation of
Urban Local Bodies in the country. These cities are spread
across 14 States.
Ministry of Urban Development is promoting Credit Rating of
cities as one of the Transformational Reforms.

WWW.INSIGHTSIAS.COM 1
INS INSIGHTS IAS PRELIMS TEST SERIES - 2017

Statement 1: Of the total 20 ratings ranging from AAA to D,


BBB- is the Investment Grade ratings and cities rated below
BBB- need to undertake necessary interventions to improve
their ratings for obtaining positive response to the Municipal
Bonds to be issued.
Q Source: PIB Releases: 29th March

2. Meghalaya plateau has a highly eroded surface and some


places are devoid of vegetation. Which of these can be a
probable reason?
a) It receives maximum rainfall from the Monsoon.
b) It is a major volcanic zone.
c) It is disturbed by frequent plate movements.
d) It is a region containing radioactive minerals.
Solution: a)
Justification: This area receives maximum rainfall from the
south west monsoon. As a result, the Meghalaya plateau has a
highly eroded surface. Cherrapunji displays a bare rocky surface
devoid of any permanent vegetation cover.
Option D: Similar to the Chotanagpur plateau, the Meghalaya
plateau is also rich in mineral resources like coal, iron ore,
sillimanite, limestone and uranium. However, this isnt a
reason behind the denuded surfaces.
Q Source: Indian Physical Geography: 11th NCERT

WWW.INSIGHTSIAS.COM 2
INS INSIGHTS IAS PRELIMS TEST SERIES - 2017

3. The data obtained from NISAR mission, a joint work of


ISRO and NASA, will be useful for which of these
applications?
1. Assessment of mangroves
2. Making Avalanche forecasts and studying Glacial Lake
Outburst Floods
3. Estimating agricultural biomass over the crop cycle
4. Measuring coastline changes and variation of winds in
coastal waters
Select the correct answer using the codes below.
a) 1, 2 and 4 only
b) 2, 3 and 4 only
c) 1 only
d) 1, 2, 3 and 4
Solution: d)
Justification: ISRO and Jet Propulsion Laboratory (JPL)/
NASA are jointly working on the development of Radar Imaging
Satellite named as NASA-ISRO Synthetic Aperture Radar
(NISAR).

The L & S band microwave data obtained from this


satellite will be useful for variety of application, which
include natural resources mapping & monitoring;
estimating agricultural biomass over full duration of crop
cycle etc.
It also includes assessing soil moisture; monitoring of
floods and oil slicks; coastal erosion, coastline changes &
variation of winds in coastal waters; assessment of
mangroves; surface deformation studies, ice sheet
collapses & dynamics etc.
These applications could help in taking measures to
minimize loss of human lives.
WWW.INSIGHTSIAS.COM 3
INS INSIGHTS IAS PRELIMS TEST SERIES - 2017

Q Source:
http://pib.nic.in/newsite/PrintRelease.aspx?relid=160166

4. Why the refugee problem remains grim across India-


Afghanistan border?
1. India has not signed an extradition treaty with
Afghanistan.
2. India shares a porous border with Afghanistan similar
to the India-Nepal border.
Which of the above is/are correct?
a) 1 only
b) 2 only
c) Both 1 and 2
d) None
Solution: d)
Justification: Statement 1: Till date, India has signed
Extradition Treaties with 47 countries/territories namely
Afghanistan, Nepal, Iran, China etc.
Besides, India has also entered into extradition arrangements
with countries such as Sweden, Singapore, Sri Lanka and
Tanzania. So, 1 is wrong.
The refugee situation remains grim mainly due to lack of a
proper refugee policy in India and the volatile situation in
Afghanistan.

WWW.INSIGHTSIAS.COM 4
INS INSIGHTS IAS PRELIMS TEST SERIES - 2017

Statement 2: India shares a porous border only with stable


neighbours nations like Nepal and Bhutan. It is not in Indias
interest to share an open border with a volatile nation like
Afghanistan.
Q Source: Improvisation: http://www.mea.gov.in/lok-
sabha.htm?dtl/28206/QUESTION+NO3419+EXTRADITION+
TREATY

5. If a Central Bank increases the Capital Adequacy Ratio


(CAR) significantly
a) It is encouraging banks to take greater risks with their
investments.
b) It mandates banks to keep a very low amount of interest
rates on their deposits.
c) It signals near zero policy rates to encourage lending by
banks.
d) It means that the banks are required to keep relatively
more capital with them for the investments they make
Solution: d)
Justification & Learning: Capital Adequacy Ratio (CAR) is
the ratio of a bank's capital in relation to its risk weighted assets
and current liabilities.

It is decided by central banks and bank regulators to


prevent commercial banks from taking excess leverage and
becoming insolvent in the process.
The Basel III norms stipulated a capital to risk weighted
assets of 8%. However, as per RBI norms, Indian
scheduled commercial banks are required to maintain a
WWW.INSIGHTSIAS.COM 5
INS INSIGHTS IAS PRELIMS TEST SERIES - 2017

CAR of 9% while Indian public sector banks are


emphasized to maintain a CAR of 12%.
Option A and D: Higher the CAR, higher the amount of capital
that banks are needed to keep with them. This discourages risky
lending and increases the capital with banks per quantum of
investment. So, A is wrong and D is correct.
Q Source: Chapter on Banking: Indian Economy: Ramesh
Singh

6. Consider the following about Cognitive Science Research


Initiative (CSRI).
1. The initiative aims to study mind and its impressions to
better understand the Indian behavioural peculiarities
and Indian languages.
2. It aims to examine the nature and origins of mental
disorders so that better learning tools and educational
paradigms can be designed.
Which of the above is/are correct?
a) 1 only
b) 2 only
c) Both 1 and 2
d) None
Solution: c)
Justification: Cognitive Science is the study of human mind
and brain, focusing on how mind represents and manipulates
knowledge and how mental representations and processes are
realized in the brain.
WWW.INSIGHTSIAS.COM 6
INS INSIGHTS IAS PRELIMS TEST SERIES - 2017

It is important to foster scientific research in


interdisciplinary field of Cognitive Science for better
understanding of Indian mind sets, languages and
cognitive disorders etc.
With this aim, the Department of Science & Technology
(DST) had initiated a highly focused programme
"Cognitive Science Research Initiative (CSRI)" in 2008.
The Cognitive Science Research Initiative facilitates a
platform to scientific community to work for better
solution of challenges related with cognitive disorders and
social issues through various psychological tools &
batteries, early diagnosis & better therapies, intervention
technologies and rehabilitation programmes.
Q Source: http://dst.gov.in/cognitive-science-research-
initiative-csri

7. Growing consciousness over the deleterious effects of


alcohol on human societies and that of plastic on our
ecology has led to their ban in many states. Which of these
states have prohibited the use of alcohol for human
consumption fully and use of plastics either partially or
fully?
1. Bihar
2. Gujarat
3. Karnataka
4. Sikkim
Select the correct answer using the codes below.
a) 1 and 2 only
b) 2, 3 and 4 only
WWW.INSIGHTSIAS.COM 7
INS INSIGHTS IAS PRELIMS TEST SERIES - 2017

c) 1 and 4 only
d) 2 and 3 only
Solution: a)
Justification: Alcohol prohibition in India is in force in the
states of Gujarat, Kerala, Bihar, Nagaland and Manipur; as well
as in the Union Territory of Lakshadweep.
States that have banned plastics include Karnataka, Bihar,
Uttar Pradesh (partial), Sikkim, HP, Goa (partial), Gujarat etc.
Around 10 states have banned plastics either partially or fully.
HP was the first state to ban plastics.
Q Source: Current Affairs:
http://timesofindia.indiatimes.com/city/bengaluru/Total-
plastic-ban-in-Karnataka/articleshow/51397198.cms
http://www.thehindu.com/news/national/karnataka/plastic-
products-banned-in-10-states-govt-to-hc/article8391018.ece
http://www.ndtv.com/india-news/chief-minister-anandiben-
patel-declares-plastic-ban-across-gujarat-1207710

8. With reference to Ancient India, which of these factors


may have contributed to the downfall of the Magadha
Empire?
1. The empire did not posses iron ore reserves which were
vital for producing weapons.
2. The empire could fall prey to invasions easily due to not
being situated near hills or surrounded by them.
Which of the above is/are correct?

WWW.INSIGHTSIAS.COM 8
INS INSIGHTS IAS PRELIMS TEST SERIES - 2017

a) 1 only
b) 2 only
c) Both 1 and 2
d) None
Solution: d)
Justification: Statement 1: Magadha enjoyed an
advantageous geographical position in the age of iron because
the richest iron deposits were situated not far away from Rajgir,
the earliest capital of Magadha.

The ready availability of rich iron ores in the


neighbourhood enabled the Magadhan princes to equip
themselves with weapons, which wore not easily available
to their rivals. So, 1 is clearly wrong.
However, Iron mines were also found in eastern MP, and
were not far from the kingdom of the Avantis with their
capital at Ujjain. Around 500 B.C. iron was certainly
forged and smelted in Ujjain making weapons of good
quality.
On account of this Avanti proved to be the most serious
competitor of Magadha for the supremacy of north India,
and Magadha took about a hundred years to subjugate
Ujjain.
Statement 2: Magadha enjoyed certain advantages which made
it informidable.

The two capitals of Magadha, the first at Rajgir and the


second at Pataliputra, were situated at very strategic
points.
Rajgir was surrounded by a group of five hills, and so it
was rendered impregnable in those days when there were
no easy means of storming citadels such as cannons

WWW.INSIGHTSIAS.COM 9
INS INSIGHTS IAS PRELIMS TEST SERIES - 2017

In the fifth century the Magadhan princes shifted their


capital from Raigir to Pataliputra which occupied a pivotal
position commanding communications on all sides.
Pataliputra was situated at the confluence of the Ganga,
the Gandak and the Son, and a fourth river called the
Sarayu.
Q Source: Ancient India: Old NCERT: RS Sharma

9. North-Eastern tribes living in the Himalayan regions


belong to which of the following races?
a) Negritoes
b) Proto-Australoids
c) Mongoloids
d) Western Brachycephals
Solution: c)
Justification & Learning: The tribes of India are classified
into different groups on the basis of geographical location,
language, race and levels of their socioeconomic development.
When we talk of racial groups, there are three broad
classifications (1) the Mongoloid, (2) the Proto-Austroloid, (3)
the Negrito.

The Mongoloid: The North-North-Eastern tribes living in


the Himalayan regions belong to the Mongoloid race.
The Australoid: The tribes of central India are mainly
Proto-Australoid in their racial ancesting.

1
WWW.INSIGHTSIAS.COM
0
INS INSIGHTS IAS PRELIMS TEST SERIES - 2017

The Negrito: A few tribes like the Kadar of the south and
the Andamanese of the Andaman Islands belong to this
race.
Q Source: Improvisation: Past year UPSC papers

10. DRAMBORA, PLATTER and PLANETS Project are


associated with tackling which of these major challenges of
the modern world?
a) Environmental Conservation
b) Digital preservation of data
c) Food Security and inter-breeding of crops
d) Earth stability at check dams
Solution: b)
Learning: it is not necessary to know the techniques of
DRAMBORA and others in detail. It is more important to
understand the context.

The explosion of digital information has caught us


unprepared to handle the threats posed by rapidly
changing technologies and digital obsolescence of
computer hardware, software, file formats and storage
media.
The possibilities of data corruption, physical damage and
disasters continue to endanger digitally encoded
information.
The obsolescence of digital content, e-records and the
evidentiary proofs can create problems in administrative,
judiciary and legislative functions in addition to loss of

1
WWW.INSIGHTSIAS.COM
1
INS INSIGHTS IAS PRELIMS TEST SERIES - 2017

valuable information, intellectual property and heritage


for future generations.
DeitY has taken early steps towards envisaging a National
Digital Preservation Program.
A system for the "audit and certification of trustworthy
digital repositories" was developed by the Consultative
Committee for Space Data Systems (CCSDS) and
published as ISO standard.
Q Source: Improvisation: http://deity.gov.in/content/digital-
preservation

11. Which of these is correct with reference to Sweat


equity shares?
a) They are issued in the Initial Public offer (IPO) by
Public Sector Enterprises
b) They are issued by a Company to its employees in view
of their intellectual property rights made available to
the company
c) They are ordinary shares purchasable only by informal
sector employees or casual labour
d) They are issued by the banks for debt restructuring of a
company to pay off a non-performing asset (NPA)
Solution: b)
Learning: These are such equity shares, which are issued by a
Company to its directors or employees at a discount or for
consideration, other than cash, for providing their know-how or
making available rights in the nature of intellectual property
rights or value additions.

1
WWW.INSIGHTSIAS.COM
2
INS INSIGHTS IAS PRELIMS TEST SERIES - 2017

The Ministry of Corporate Affairs vide a 2016 notification has


amended Companies (Share Capital and Debentures) Rules,
2014 to allow a startup company to issue sweat equity shares
not exceeding fifty per cent of its paid up capital upto five years
from the date of its incorporation or registration.
Q Source: PIB Releases: Ministry of Corporate Affairs:
March 2017

12. The velocity of sound in air increases with


1. Increase in air temperature
2. Decrease in air density
3. Decrease in humidity of air
Select the correct answer using the codes below.
a) 1 only
b) 1 and 2 only
c) 3 only
d) 2 and 3 only
Solution: b)
Justification: The speed of sound in a medium depends upon
the following two factors: (i) the elasticity E of the medium and
(ii) the density of the medium. The speed of sound in a
medium is given by the formula
v = (E/), where E is the modulus of elasticity of the medium
and the density of the medium

1
WWW.INSIGHTSIAS.COM
3
INS INSIGHTS IAS PRELIMS TEST SERIES - 2017

Statement 1: The speed of sound in a gas increases with


increase in temperature of the gas. v T, where T is the
temperature of the gas in kelvin.
Molecules at higher temperatures have more energy, thus they
can vibrate faster. Since the molecules vibrate faster, sound
waves can travel more quickly.
Statement 2: The speed of sound is inversely proportional to
the square root of density of the gas. It means it increases with
the decrease in density.
Statement 3: The speed of sound in air increases with the
increase in humidity in the air. Sound travels faster in humid
air than dry air.
Q Source: General Science based Concepts

13. Satya Narayan Goenka had been awarded Padma


Bhushan by the Government of India for his contribution
to the spread of the Vipassana technique over India and
the world. What is the technique about?
a) Detoxifying the body through naturopathy
b) Observing reality objectively through meditation
c) Separating the ethnic groups from non-ethnic groups in
a large tribal population
d) Understanding the cycles of nature through
astronomical calculations
Solution: b)

1
WWW.INSIGHTSIAS.COM
4
INS INSIGHTS IAS PRELIMS TEST SERIES - 2017

Learning: It is a Buddhist technique that has been taught


since the last 2500 years to monks and practitioners.

The Vipassana course start with observation of natural


(i.e. not controlled) breath, which allows the mind to
become concentrated, a practice called Anapana.
This concentration prepares one for the Vipassana practice
itself which, in this tradition, involves observing bodily
sensations with equanimity and becoming progressively
more aware of the interconnection between mind and
body.
Goenka was a Burmese-Indian teacher of Vipassana
meditation. His teaching was notable for emphasizing that
the Buddha's path to liberation was non-sectarian,
universal, and scientific in character.
Goenka was an invited speaker at the Millennium World
Peace Summit of Religious and Spiritual Leaders in 2000
at the United Nations in New York.
Q Source: http://www.ndtv.com/india-news/arvind-kejriwal-
reaches-dharamsala-for-vipassana-course-1438809
https://www.theatlantic.com/health/archive/2017/03/north-
korea-veganism/520578/

14. Consider the following about National Mission on


Libraries (NML).
1. It will create a National Virtual Library of India to
facilitate a comprehensive database on digital resources.

1
WWW.INSIGHTSIAS.COM
5
INS INSIGHTS IAS PRELIMS TEST SERIES - 2017

2. Under the mission, all public libraries will be replaced


by privately owned and funded libraries in phases
across India.
Which of the above is/are correct?
a) 1 only
b) 2 only
c) Both 1 and 2
d) None
Solution: a)
Justification: The scheme consists of four components:
Creation of National Virtual Library of India (NVLI), Setting up
of NML Model Libraries, Quantitative & Qualitative Survey of
Libraries and Capacity Building in existing libraries
(personnel).
It has been set up by Ministry of Culture, Government of India
in pursuance of National Knowledge Commission
recommendations for sustained attention for development of
Libraries and Information Science Sector.
Q Source: http://www.nmlindia.nic.in/pages/display/38-
national-virtual-library-of-india-(nvli)
http://www.nmlindia.nic.in/

15. Consider the following about Kala-azar.

1
WWW.INSIGHTSIAS.COM
6
INS INSIGHTS IAS PRELIMS TEST SERIES - 2017

1. The disease occurs in tropical as well as temperate


climatic regions.
2. It is transmitted only through the mode of water, air or
human contact.
3. There is no known treatment for Kala-azar.
Select the correct answer using the codes below.
a) 1 and 2 only
b) 1 only
c) 2 and 3 only
d) 3 only
Solution: b)
Justification: This disease is the second-largest parasitic
killer in the world (after malaria)
Statement 1: The vectors of kala-azar are found in tropical or
temperate regions throughout the world.
Statement 2: Indian Kala-azar has a unique epidemiological
feature of being Anthroponotic; human is the only known
reservoir of infection. Sandfly of genus Phlebotomus argentipes
are the only known vectors of kala-azar in India
Statement 3: Kala-azar Drugs are available in India as per the
Drug Policy under Kala-azar Elimination Programme as per
recommendations of Expert Committee (2000).
Q Source: Frequently in news:
http://www.ptinews.com/news/8547809_Health-policy-
focuses-on-expanding-access--quality--Secy.html

1
WWW.INSIGHTSIAS.COM
7
INS INSIGHTS IAS PRELIMS TEST SERIES - 2017

16. Consider the following statements about Jorwe


Culture.
1. It is the first known culture to have abandoned symbolic
burial practices instead adopting fire rituals for last
rites.
2. It did not practice pottery or agriculture as Jorwe
ancestors considered earth a sacred entity to be
worshipped and conserved.
Which of the above is/are correct?
a) 1 only
b) 2 only
c) Both 1 and 2
d) None
Solution: d)
Justification: Pravara-Godavari Valley was the nuclear zone
of Jorwe Culture. The main sites of Jorwe Culture are
Daimabad, Inamgaon, Jorwe and Nevasa.
Statement 1: The culture was characterised by a distinct type of
painted pottery, a blade-flake industry of chalcedony, as well as
tools and ornaments of copper. However, due to the scarcity of
the metal, copper was used sparingly.

Their mixed economy was based on agriculture, stock-


raising, hunting and fishing. They cultivated a variety of
crops, including cereals.
They practised crop rotation because it gave them the
facility of irrigation - clear evidence of that has been
unearthed at Inamgaon, near Pune.
1
WWW.INSIGHTSIAS.COM
8
INS INSIGHTS IAS PRELIMS TEST SERIES - 2017

Statement 2: At Daimabad and Inamgaon, one gets


archaelogical evidence of symbolic burial

They believed in life after death, and therefore interred the


dead inside the house under the floor.
Children were buried in two urns that were joined mouth-
to-mouth and set horizontally in the pit, while adults were
placed in a supine position with the head towards the
North.
Q Source: Improvisation: Ancient India: Old NCERT: RS
Sharma

17. Which of these states is the first in the country to


adopt Self Help Groups (SHG) based financial inclusion
model to extend banking services in the unbanked areas
through them?
a) Himachal Pradesh
b) Jharkhand
c) Telangana
d) Odisha
Solution: d)
Learning: In this regard, state government has inked
agreement with the State Bank of India (SBI). The SHGs
formed under the Odisha Livelihood Mission (OLM) will be
eligible to provide banking services.

The agreement assumes significant as 70% of the gram


panchayats in the State do not have brick and mortar bank
branches.
1
WWW.INSIGHTSIAS.COM
9
INS INSIGHTS IAS PRELIMS TEST SERIES - 2017

As per the agreement, OLM will function as corporate


agent and around 3 lakh SHGs operating under it will be
the banking correspondents (BCs).
In the first phase SHGs will be engaged as Banking
Correspondents (BCs) in around 1000 remote GPs in
scheduled areas having no banking facilities.
Subsequently entire State will be covered under the
financial inclusion programme.
Q Source: Odisha Government Website

18. As noted in Arthashatra, adhyakshas were appointed


to
a) Regulate the economic activities of the state
b) Lead military expansion of the empire
c) Provide sound policy advice to the Emperor
d) Recruit officers for administering the empire
Solution: a)
Justification: The Mauryan state appointed 27
superintendents (adhyakshas) mostly to regulate the economic
activities of the state. They controlled and regulated agriculture,
trade and commerce, weights and measures, crafts such as
weaving and spinning, mining, and so on.
Learning: The state also provided irrigation facilities and
regulated water supply for the benefit of agriculturists.
Megasthenes informs us that in the Maurya empire the officer
measured the land as in Egypt and inspected the channels
through which water was distributed into fields.

2
WWW.INSIGHTSIAS.COM
0
INS INSIGHTS IAS PRELIMS TEST SERIES - 2017

Q Source: Ancient India: Old NCERT: RS Sharma

19. When the Montagu report of 1918 was made public,


there was a divide in the Congress over it causing a schism
in the Congress leading to the formation of?
a) National Liberal Federation of India
b) Revolutionary Socialist Party
c) Congress Socialist Party
d) Communist Party of India
Solution: a)
Justification: Work by elimination.
Option D: Communist Party of India in 1925, much later after
these episodes happened. So, D is incorrect.
Option C: Congress Socialist Party, or (CSP), was a left-wing
group within the Congress. But, it was formed in 1934. So, C is
incorrect.
Option B: Revolutionary Socialist Party (RSP) was founded in
1940 and has its roots in the Bengali liberation movement. So,
B is incorrect as well.
Learning: The moderates welcomed the report while the
extremists opposed it. This led to a schism in the Congress with
moderate leaders forming the "National Liberal Federation of
India" in 1919.

Its most prominent leaders were Tej Bahadur Sapru, V. S.


Srinivasa Sastri and M. R. Jayakar.

2
WWW.INSIGHTSIAS.COM
1
INS INSIGHTS IAS PRELIMS TEST SERIES - 2017

Tej Bahadur Sapru emerged as the most important leader


among the Liberals. During the agitation against the
Simon Commission, he launched the idea of an all-parties
conference in India to prepare an agreed constitutional
scheme.
This resulted in the "Nehru Report" which proposed a
constitution and persuaded the new Labour Government
in Britain to offer India a Round Table Conference.
Q Source: Improvisation: Past year UPSC papers

20. Consider the following about the Society for


Worldwide Interbank Financial Telecommunication
(SWIFT).
1. It is a messaging network that financial institutions use
to securely transmit information and instructions
through a standardized system of codes.
2. Indian Government has recently authorized SWIFT to
hold foreign currency funds and securities to better
manage client accounts.
Which of the above is/are correct?
a) 1 only
b) 2 only
c) Both 1 and 2
d) None
Solution: a)

2
WWW.INSIGHTSIAS.COM
2
INS INSIGHTS IAS PRELIMS TEST SERIES - 2017

Justification: Statement 1: SWIFT assigns each financial


organization a unique code that has either eight characters or 11
characters. The code is called interchangeably the bank
identifier code (BIC).

Prior to SWIFT, Telex was the only available means of


message confirmation for international funds transfer.
Telex was hampered by low speed, security concerns, and
a free message format--in other words, Telex did not have
a unified system of codes like SWIFT to name banks and
describe transactions.
Statement 2: It is only a messaging systemSWIFT does not
hold any funds or securities, nor does it manage client accounts.

SWIFT India Domestic Services last year (2016) rolled out


services to provide harmonised exchange of structured
financial information between banks, the Reserve Bank of
India, stock exchanges, clearing houses, corporations, and
their customers.
Learning: Assume a customer of a Bank of America branch in
New York wants to send money to his friend who banks at the
SBI branch in India.

The New Yorker can walk into his Bank of America branch
with his friends account number and SBI, India unique
SWIFT code for its India branch.
Bank of America will send a payment transfer SWIFT
message to the SBI branch over the secure SWIFT
network.
Once SBI receives the SWIFT message about the incoming
payment, it will clear and credit the money to the friends
account.

2
WWW.INSIGHTSIAS.COM
3
INS INSIGHTS IAS PRELIMS TEST SERIES - 2017

Q Source: http://www.thehindubusinessline.com/money-
and-banking/swift-india-rolls-out-financial-information-
sharing-service/article8314724.ece

21. Which of the following countries was admitted as the


latest member of North Atlantic Treaty Organization
(NATO)?
a) Montenegro
b) Canada
c) Switzerland
d) None of the above
Solution: d)
Learning: US Senate recently voted with near unanimity to
approve Montenegro as 29th member of NATO. So far, 25 other
NATO members have ratified Montenegros accession, a small
country which is being seen as a geostrategic ally. However,
Spain and Netherlands are yet to ratify its membership. So, D is
the answer.
Learning: The NATO is an intergovernmental military alliance
based on the North Atlantic Treaty signed in 1949. It consists of
28 independent member countries across North America and
Europe.

It is based on a system of collective defence whereby its


member states agree to mutual defence in response to an
attack by any external party.

2
WWW.INSIGHTSIAS.COM
4
INS INSIGHTS IAS PRELIMS TEST SERIES - 2017

NATOs headquarters are located in Brussels, Belgium. Its


headquarters of Allied Command Operations is near
Mons. The combined military spending of all NATO
members constitutes over 70% of the global defence
spending.
Q Source: Current Affairs: International organizations

22. The Self-respect Movement of Periyar E.V.R.


focussed on
1. Eliminating the Purdah system for women in certain
communities
2. Providing public health and sanitation facilities for the
poorest families
3. Getting social justice for the untouchables
Select the correct answer using the codes below.
a) 1 and 2 only
b) 3 only
c) 2 and 3 only
d) 1 only
Solution: b)
Justification: The movement began in 20th century in the
Madras Presidency.

In order to eradicate this evil practice individual and


institutional efforts were made. These movements were
directed mainly in removing the disabilities suffered by
Harijans in regard to drawing of water from public wells,
getting entry into temples and admission into schools.
2
WWW.INSIGHTSIAS.COM
5
INS INSIGHTS IAS PRELIMS TEST SERIES - 2017

In 1970 the UNESCO praised and adorned Periyar with


the title Socrates of South Asia.
A number of political parties in Tamil Nadu, such as
Dravida Munnetra Kazhagam (DMK) and All India Anna
Dravida Munnetra Kazhagam (AIADMK) owe their origins
to the Self-respect movement.
Q Source: TN Textbook: 12th Standard: Modern History

23. Why do Himalayan Rivers form ox-bow lakes, but not


Peninsular Rivers?
1. Peninsular rivers do not meander, unlike Himalayan
Rivers which meander considerably increasing chances
of lake formation.
2. Peninsular rivers flow in shallow valleys with hard
terrain, unlike Himalayan Rivers where valleys are deep
and terrain is softer.
Which of the above is/are correct?
a) 1 only
b) 2 only
c) Both 1 and 2
d) None
Solution: c)
Justification: Himalayan Rivers have long courses and are
engaged in erosional and depositional activities. Passing
through the mountains, they make deep gorges. They bring
huge quantities of silt and sediment.

2
WWW.INSIGHTSIAS.COM
6
INS INSIGHTS IAS PRELIMS TEST SERIES - 2017

In the middle and lower courses they make meanders, ox-


bow lakes, flood plains and deltas.
The Peninsular Rivers flow through broad shallow valleys.
Their course is more or less short and straight.
Due to hard rocks and lack of silt and sediment they do not
form meanders and ox-bow lakes. Most of them rise in the
Western Ghats flow eastwards and fall into the Bay of
Bengal, forming fertile deltas.
Q Source: ICSE Geography Textbook: Class IX

24. India is a net exporter of


1. Crude Oil
2. Electricity
3. Coking coal
Select the correct answer using the codes below.
a) 1 only
b) 2 only
c) 2 and 3 only
d) 1, 2 and 3
Solution: b)
Justification: Statement 1: India imports nearly 80% of its
crude oil requirements. It however exports a large quantum of
processed oil products.

2
WWW.INSIGHTSIAS.COM
7
INS INSIGHTS IAS PRELIMS TEST SERIES - 2017

Statement 2: Ever since the cross border trade of electricity


started in mid-Eighties, India has been importing power from
Bhutan and marginally exporting to Nepal.

But, as per Central Electricity Authority, for the first time


India has turned around from a net importer of electricity
to Net Exporter of electricity during 2016-17.
India has exported to Nepal, Bangladesh and Myanmar.
Export to Nepal and Bangladesh increased 2.5 and 2.8
times respectively in last three years.
Statement 3: Coking coal imports by Indian steel makers are
pegged at 50 million tonnes in 2016-17.
Q Source: Improvisation: Several Chapters of India Yearbook
2017

25. The Honey Bee Network operating in India works to


a) Connect local internet server grids to international
server centres
b) Increase awareness of the economical value of natural
pollination practices in agricultural farms
c) Uncover the hidden innovative talent in India
d) Conserve and preserve the genetics of important insect
species of India
Solution: c)

2
WWW.INSIGHTSIAS.COM
8
INS INSIGHTS IAS PRELIMS TEST SERIES - 2017

Learning: This question has been asked in the context of the


Festival of Innovation (FOIN) 2017.

FOIN is a week-long annual event organised by the


Presidents Secretariat and is assisted by the National
Innovation Foundation (NIF), an autonomous institution
under the Department of Science and Technology (DST),
and Society for Research and Initiatives for Sustainable
Technologies and Institutions (SRISTI), part of the Honey
Bee Network.
The Honey Bee Network, a social movement to uncover
the hidden innovative talent, started 25 years ago and has
made a small contribution towards recognition, respect
and reward for the unsung heroes of our society which will
help in making India creative, compassionate and a
collaborative society.
These institutions (in the link)
http://www.sristi.org/cms/en/our_network form part of
the network.
Q Source: http://nif.org.in/foin-2017

26. The Cabinet has approved the clause of Permanent


Residency Status (PRS) in India for which of these
categories of persons?
a) Non-Resident Indians (NRIs) who frequently visit India
b) Foreign investors investing a certain threshold in India
c) Victims of war crimes and refugees from friendly
nations
d) Persecuted minorities of Indian origin living abroad

2
WWW.INSIGHTSIAS.COM
9
INS INSIGHTS IAS PRELIMS TEST SERIES - 2017

Solution: b)
Learning: The scheme is expected to encourage foreign
investment in India and facilitate Make in India Programme.
Under the Scheme, suitable provisions will be incorporated in
the Visa Manual to provide for the grant of PRS to foreign
investors.

Permanent Residency Status will be granted for a period of


10 years initially with multiple entry facility, which can be
renewed for another 10 years.
PRS will serve as a multiple entry visa without any stay
stipulation and PRS holders will be exempted from the
registration requirements. PRS holders will be allowed to
purchase one residential property for dwelling purpose.
The spouse/ dependents of the PRS holder will be allowed
to take up employment in private sector (in relaxation to
salary stipulations for Employment Visa) and undertake
studies in India.
This status will be subject to the relevant conditions in the
FDI Policy notified by the Central Government from time
to time.
Q Source: http://www.business-
standard.com/article/economy-policy/invest-rs-10-cr-be-a-
permanent-resident-of-india-116083101327_1.html

27. Match the following major intelligence agencies with


the nations they work for.
1. Mossad: Iraq
2. MI6: United Kingdom
3
WWW.INSIGHTSIAS.COM
0
INS INSIGHTS IAS PRELIMS TEST SERIES - 2017

3. CIA: France
4. GRU: Russia
Select the correct answer using the codes below.
a) 1 only
b) 2 and 4 only
c) 1, 3 and 4 only
d) 2, 3 and 4 only
Solution: b)
Justification: Statement 1: It is one of the most sophisticated
and advanced intelligence agency of Israel, and it is often
believed that it is because of Mossad that Israel is still able to
hold its ground.
Statement 2: The agency has been in operation even before the
world war I. The agency is often considered to be an important
reason Britains victory in the world wars.
Statement 3: It is USAs chief intelligence agency.
Statement 4: It is Russias main intelligence agency. Before the
fall of USSR, the countrys main spy agency was the KGB. The
GRU was formed as Soviet military intelligence organization
and engaged only in foreign intelligence activities.
Q Source: Touching hitherto uncovered areas

28. Which of the following is NOT a water-borne disease?


a) Cholera
b) Diarrhoea
3
WWW.INSIGHTSIAS.COM
1
INS INSIGHTS IAS PRELIMS TEST SERIES - 2017

c) Leprosy
d) Typhoid
Solution: c)
Justification & Learning: Option C: Leprosy is caused by a
slow-growing type of bacteria called Mycobacterium leprae. M.
leprae are spread person to person by nasal secretions or
droplets. However, it is not highly infectious.
Option D: Typhoid fever is caused by Salmonella typhi due to
ingestion of water contaminated with feces of an infected
person.
Option A: Cholera is spread due to drinking water
contaminated with the bacterium Vibrio cholera.
Option B: Rotavirus is a common cause of childhood diarrhea.
Bacterial infections due to Salmonella or Escherichia coli,
among others, are also common. Infection is spread through
contaminated food or drinking-water.
Q Source: 8th NCERT: Science

29. Which of the following provisions of the


Constitutions of India need the ratification by the
legislatures of not less than one-half of the States to effect
amendment?
1. Extent of the legislative powers of the Union and the
States
2. Powers of the Supreme Court and High Courts

3
WWW.INSIGHTSIAS.COM
2
INS INSIGHTS IAS PRELIMS TEST SERIES - 2017

3. Acquisition of a new territory by the Government of


India
Select the correct answer using the codes below.
a) 1 and 2 only
b) 2 and 3 only
c) 3 only
d) 2 only
Solution: a)
Justification: Those provisions of the Constitution which are
related to the federal structure of the polity can be amended by
a special majority of the Parliament and also with the consent
of half of the state legislatures by a simple majority.
There I no time limit within which the states should give their
consent to the bill.
The following provisions can be amended in this way:

Election of the President and its manner.


Extent of the executive power of the Union and the states.
Supreme Court and high courts.
Distribution of legislative powers between the Union and
the states.
Any of the lists in the Seventh Schedule.
Representation of states in Parliament.
Power of Parliament to amend the Constitution and its
procedure
Any of the Lists in the 7th Schedule
Q Source: Chapter on Amendment of the Constitution: Indian
Polity: M Laxmikanth

3
WWW.INSIGHTSIAS.COM
3
INS INSIGHTS IAS PRELIMS TEST SERIES - 2017

30. Consider the following statements.


1. India is a party to Multilateral Convention on Mutual
Administrative Assistance in Tax Matters.
2. A Bilateral Investment Agreement (BIT), to which India
is a party, must be approved by the WTO to be
considered valid and effective.
Which of the above is/are correct?
a) 1 only
b) 2 only
c) Both 1 and 2
d) None
Solution: a)
Justification: Statement 1: It was developed jointly by the
OECD and the Council of Europe in 1988.

The Convention represents a wide range of countries,


including all G20, BRIICS and OECD countries, financial
centres and several developing countries.
India is among the 98 countries and jurisdictions that
have already joined the Convention.
The convention regulates information exchange between
states parties on the exchange of information regarding
tax matters.
Panama has agreed to sign a multilateral tax treaty, which
the Indian agencies believe will help them expedite
investigations into the Panama papers.

3
WWW.INSIGHTSIAS.COM
4
INS INSIGHTS IAS PRELIMS TEST SERIES - 2017

Statement 2: A bilateral investment treaty (BIT) is an


agreement establishing the terms and conditions for private
investment by nationals and companies of one state in another
state (FDI).

It does not need to be approved by WTO, but investors can


seek recourse to WTO (under dispute resolution
mechanism) claiming violation of core WTO principles
such as national treatment.
BIT increases the comfort level and boosts the confidence
of investors.
Q Source:
http://www.thehindubusinessline.com/economy/policy/top-
mea-official-urges-panama-to-expedite-tax-information-
exchange-pact/article9508331.ece

31. Air-conditioners will turn ineffective in cooling a


spatial unit if the temperature of fluids does not depend on
its
a) Volume or pressure
b) Its suspended colloids
c) Molecular mass
d) Velocity or speed
Solution: a)

3
WWW.INSIGHTSIAS.COM
5
INS INSIGHTS IAS PRELIMS TEST SERIES - 2017

Justification: In an air-conditioner, heat is extracted from the


room-air at the evaporator coils and is rejected out at the
condenser coils.
This heat exchange takes place via a fluid that can easily convert
from a gas to a liquid and back again.
The fluid arrives at the compressor as a cool, low-pressure gas.
The compressor squeezes the fluid. This packs the molecule of
the fluid closer together. The closer the molecules are together,
the higher its energy and its temperature. (validating option A).
The working fluid leaves the compressor as a hot, high pressure
gas and flows into the condenser.

If fluid temperature does not depend on air pressure or volume,


it will be impossible to cool an air parcel by the fluid (through
the expansion or contraction of the fluid). The heat exchange
mechanism will simply not work.
3
WWW.INSIGHTSIAS.COM
6
INS INSIGHTS IAS PRELIMS TEST SERIES - 2017

Q Source: General Science Concepts

32. The World Conference on Environment held recently


in New Delhi was organized by the
a) Convention on Biological Diversity (CBD)
b) World Economic Forum (WEF)
c) International Union for Conservation of Nature (IUCN)
d) National Green Tribunal (NGT), India
Solution: d)
Learning: The President of India inaugurated the World
Conference on Environment organized by the National Green
Tribunal in New Delhi in March 2017.
Some of the alarming facts highlighted at the conference were:

Environmental factors are responsible for an estimated


24% of the global burden of disease in terms of healthy life
years lost and 23 % of all deaths.
Children are the worst sufferers of the adverse impact of
environmental diseases. 24% of all deaths under the age of
15 are due to diarrhoea, malaria and respiratory diseases
all of which are environment related.
19% of the deaths caused by Cancer worldwide are
attributed to carcinogens that unmindful industrialization
produces.
Q Source: PIB Releases: Office of President: March 2017

3
WWW.INSIGHTSIAS.COM
7
INS INSIGHTS IAS PRELIMS TEST SERIES - 2017

33. Spread at Vindhya hills, the density of the Tiger


population here is the highest known in India. Hosting tall
grasslands to thick Sal forests, this national park is
a) Jim Corbett
b) Bandhavgarh
c) Orang
d) Pambadum
Solution: b)
Learning: Located in MP, Bandhavgarh has one of the highest
density of Bengal tigers known in the world, and highest in
India, and is home to some famous named tigers, including
white tigers.
The Bandhavgarh Fort nearby is thought to be some 2000
years old, and there are references to it in the ancient
books, the Narad-Panch Ratra and the Shiva Purana.
The name Bandhavgarh has been derived from the most
prominent hillock of the area of Umaria.
Due to varied topography, the Bandhavgarh national park
provides ample opportunity to spot the majestic Indian
tiger and some rarely seen animals like leopard and sloth
bear. Due to high wildlife sighting it is becoming popular
amongst tourists visiting India.
Q Source: National Parks of India: MP Government Website

34. Consider the following about the International


Chambers of Commerce (ICC).
1. It is an inter-governmental organization chaired by the
member nations Commerce Ministers in rotation.
3
WWW.INSIGHTSIAS.COM
8
INS INSIGHTS IAS PRELIMS TEST SERIES - 2017

2. It has been granted general consultative status with the


United Nations Economic and Social Council.
Which of the above is/are correct?
a) 1 only
b) 2 only
c) Both 1 and 2
d) None
Solution: b)
Justification: ICC is the worlds largest business organisation
that was founded in 1919 headquartered in Paris, France.

It represents the private-sectors views to national


governments and intergovernmental bodies around the
world. It is not an inter-governmental body itself. So, 1 is
wrong.
It was the first international organization to be granted
general consultative status with the United Nations
Economic and Social Council.
Learning: ICC had elected founder and chairman of Bharti
Enterprises, Sunil Bharti Mittal, as its chairman in 2016 session
in Brazil. He is the third Indian business leader to hold this
position in ICC's near 100 years history.
Q Source: http://www.business-
standard.com/article/companies/sunil-bharti-mittal-elected-
as-chairman-of-international-chamber-of-commerce-
116061600167_1.html

3
WWW.INSIGHTSIAS.COM
9
INS INSIGHTS IAS PRELIMS TEST SERIES - 2017

35. Consider the following about Nari Adalat, a novel


intervention of the Government of Gujarat.
1. These courts are recognized by the State as a legal
forum with a decree of civil court.
2. Only women jurists dispense cases in Nari Adalat.
Which of the above is/are correct?
a) 1 only
b) 2 only
c) Both 1 and 2
d) None
Solution: b)
Justification: The concept devised By women for women -
the Nari Adalats is operational for legal justice in over 19
regions in Gujarat.

Women jurists dispense justice in womens cases of


divorce, abandonment, violence, rape and dowry
demands.
These courts are set up for women empowerment and
gender justice.
These courts are not recognized by the State as a legal
forum. So, 1 is incorrect. However, the autonomous hybrid
institutions are para legal authority who solve women
cases faster than judicial courts.
These courts are helping rural women overcome problems
encountered in the normal judicial system. Inaccessibility,
cost, time, unfamiliarity with legal procedures, inadequate
resources, and a traditional disregard of the needs of
women all solutions get speedy, efficacy, and cost
effective.

4
WWW.INSIGHTSIAS.COM
0
INS INSIGHTS IAS PRELIMS TEST SERIES - 2017

Q Source:
http://www.gujaratindia.com/initiatives/initiatives.htm#Gaur
av_Nari_Niti___Womens_Pride,_Gender_Equality
http://indianexpress.com/article/cities/ahmedabad/mahila-
samakhya-gujarat-mahila-samakhya-mahila-samakhya-
programmegujarat-govt-decides-to-continue-mahila-
samakhya-under-education-department-2761257/

36. Consider the following about the Indian Tsunami


Early Warning Centre (ITEWC).
1. The centre is capable of detecting tsunami-genic
earthquakes occurring in the Indian Ocean as well as in
the Global Oceans.
2. The centre provides tsunami warnings and related
services to all countries in the Indian Ocean Rim.
Which of the above is/are correct?
a) 1 only
b) 2 only
c) Both 1 and 2
d) None
Solution: c)
Justification: Statement 1: The ITEWC consists of national
and international observational network of seismic stations, sea
level gauge stations and tsunami buoys around the Indian and
Pacific Oceans.

4
WWW.INSIGHTSIAS.COM
1
INS INSIGHTS IAS PRELIMS TEST SERIES - 2017

Data from approximately four hundred seismometers is


being received in real-time and processed automatically to
detect an earthquake of magnitude 4.0 and above
anywhere on the globe.
Statement 2: ITEWC has been designated as one of the
Regional Tsunami Service Providers for the entire Indian Ocean
Region by the Intergovernmental Oceanographic Commission
(IOC) of UNESCO in 2011.

Since then, ITEWC is providing tsunami warnings and


related services to all countries in the Indian Ocean Rim
(24 Countries) beyond fully serving the Indias coastline /
Islands.
The centre can detect tsunami generating earthquakes
within 10 minutes of their occurrence and disseminates
the advisories to the concerned authorities within 20
minutes through email, fax, SMS, Global
Telecommunication System (GTS) and website.
Q Source: PIB releases: 29th March

37. There are six Darshan Shashtras, which is also known


as Shat Darshan. Which of these do NOT form its part?
1. Nyaya
2. Vaisheshika
3. Mimansa
4. Sulabha
Select the correct answer using the codes below.
a) 1 and 2 only

4
WWW.INSIGHTSIAS.COM
2
INS INSIGHTS IAS PRELIMS TEST SERIES - 2017

b) 3 only
c) 4 only
d) 1, 2 and 4 only
Solution: c)
Justification: Some common principles of the six Darshans
(which also include Yoga, Samkhya and Vedanta) are:

By the freedom, from the three types of sorrows, Moksha


(Salvation) is obtained.
God, Soul & Prakriti are the eternal causes of the creation
of the universe.
Nonexistence of Existent objects & existence of
nonexistent objects is extremely impossible.
Vedas are self-proven, because it is gospel-word of the
God.
Soul is different from the body, mind etc, and
inconsequential.
God is different from the soul, omnipresent, infinite
intellect, omnipotent etc.,
According to the God's discipline & deeds done by the
soul, the soul repeal the results of the deeds.
The visible universe is the product of the Prakriti.
The reason of the bondage of the soul is its ignorance.
Q Source: Hindu philosophy: Ancient India

38. There is always significant fluctuation in the area


under jute cultivation and its production in India. This can
be attributed to
4
WWW.INSIGHTSIAS.COM
3
INS INSIGHTS IAS PRELIMS TEST SERIES - 2017

1. Fluctuation in average rainfall during the sowing season


2. Average price of Jute and competing crops realized in
last season
Which of the above is/are correct?
a) 1 only
b) 2 only
c) Both 1 and 2
d) None
Solution: c)
Justification: The year to year fluctuations arise out of three
factors namely, (i) fluctuation in rainfall during the sowing
season (nearly 80% jute cultivation is rainfed), (ii) the average
raw jute prices realized during the previous jute season, and
(iii) the returns realized from competing crops during the
previous season.
For e.g. a significant area under jute used to compete with
paddy during the same season. Hence, year to year fluctuations
in the prices of jute relative to the prices of paddy would
generally influence the relative allocation of land between the
two crops.
Q Source: http://www.business-standard.com/article/pti-
stories/irani-singh-discuss-steps-to-boost-jute-output-
117032201270_1.html

39. In the context of Medieval India, what were zawabits?


a) Regulations made by sultan to supplement Muslim law
4
WWW.INSIGHTSIAS.COM
4
INS INSIGHTS IAS PRELIMS TEST SERIES - 2017

b) Spies recruited by the state to intervene in the affairs of


neighbour empires
c) Market regulator of commodities, trade and commerce
within the empire
d) Superintendent of religious institutions and trusts
Solution: a)
Learning: The sultans had to supplement the Muslim law by
framing their own regulations (zawabit). Alauddin Khalji told
the leading qazi of the city that he did not know what was lawful
or unlawful but framed laws according to the needs of the state.

This is why the historian Barani refused to consider the


state in India as truly Islamic, but one based on worldly or
secular considerations (jahandari).
As for the Hindu subjects, from the time of the Arab
invasion of Sindh, they had been given the status of
zimmis or protected people, i.e. those who accepted the
Muslim rule and agreed to its regulations.
Q Source: Medieval India: Old NCERT: Satish Chandra

40. Which of these languages of India have been declared


as Critically Endangered by UNESCO?
1. Apatani
2. Baghati
3. Sherpa
4. Shompen
Select the correct answer using the codes below.
a) 1 and 3 only
4
WWW.INSIGHTSIAS.COM
5
INS INSIGHTS IAS PRELIMS TEST SERIES - 2017

b) 2 and 4 only
c) 1, 2 and 3 only
d) 2, 3 and 4 only
Solution: b)
Justification: List of all such languages can be found here
http://pib.nic.in/newsite/PrintRelease.aspx?relid=108207
An endangered language is a language that is at a risk of falling
out of use, generally because it has few surviving speakers. If it
loses all of its native speakers, it becomes an extinct language.

UNESCO defines four levels of language endangerment


between "safe" (not endangered) and "extinct":
Vulnerable, Definitely endangered, Severely endangered
and Critically endangered
The Government of India has initiated a Scheme known as
Protection and Preservation of Endangered Languages of
India.
Under this Scheme, the Central Institute of Indian
Languages (CIIL), Mysore works on protection,
preservation and documentation of all the mother
tongues/ languages of India spoken by less than 10,000
speakers keeping in mind the degree of endangerment and
reduction in the domains of usage.
Q Source:
http://pib.nic.in/newsite/PrintRelease.aspx?relid=108207

41. The programme SATYAM by the Government of


India aims to
4
WWW.INSIGHTSIAS.COM
6
INS INSIGHTS IAS PRELIMS TEST SERIES - 2017

1. Strengthen research in the areas of yoga and meditation


2. Preserve literature in select ethnic languages in the
Traditional Digital Knowledge Library (TKDL)
Which of the above is/are correct?
a) 1 only
b) 2 only
c) Both 1 and 2
d) None
Solution: a)
Justification: SATYAM-Science and Technology of Yoga and
Meditation has been launched by DST, GoI to strengthen
research in the areas of yoga and meditation.
An objective of SATYAM is to harness knowledge obtained in
academic institutions and other related agencies for finding
Science & Technology-led solutions that would enable us to
cope with stress and strain associated with fast changing social,
economic, environmental and professional circumstances.
The programme will encourage research in two major thematic
areas:

Investigations on the effect of Yoga and meditation on


physical and mental health, and
Investigations on the effect of Yoga & meditation on the
body, brain and mind in terms of basic processes
occurring therein.
Q Source: http://www.dst.gov.in/science-and-technology-
yoga-and-meditation

4
WWW.INSIGHTSIAS.COM
7
INS INSIGHTS IAS PRELIMS TEST SERIES - 2017

42. Seed is not required for every plant to reproduce.


Which of the following plants grow through spores?
1. Fern
2. Fig
3. Mosses
4. Pine
Select the correct answer using the codes below.
a) 1, 2 and 3 only
b) 3 and 4 only
c) 1 and 3 only
d) 2 and 4 only
Solution: c)
Learning: Seeds are not the only means of reproduction.
Some plants create offshoots of themselves in the form of
bulbs, tubers, corms, spores or rhizomes that can grow into
new plants.

Ferns typically produce large spores on fertile leaves


called. The spores are released and germinate to produce.
So, 1 is correct.
Figs and pine reproduce through seed. Although
commonly referred to as a fruit, the fig is actually the scion
of the tree, known as a false fruit or multiple fruit, in
which the flowers and seeds are borne.
Q Source: Improvisation: Past year UPSC papers

4
WWW.INSIGHTSIAS.COM
8
INS INSIGHTS IAS PRELIMS TEST SERIES - 2017

43. Consider the following statements.


Assertion (A): The 1857 mutiny was confined to Northern
India, and did not spread to either Southern or eastern
India.
Reason (R): The Doctrine of Paramountcy was not in
operation in Northern India.
In the context of the above, which of these is correct?
a) A is correct, and R is an appropriate explanation of A.
b) A is correct, but R is not an appropriate explanation of
A.
c) A is correct, but R is incorrect.
d) Both A and R are incorrect.
Solution: d)
Justification: While the Southern India did not participate as
enthusiastically as the Northern regions did, the participation
of eastern region was important. For e.g. centres like
Barrackpur and Kolkata were major uprising centres.

4
WWW.INSIGHTSIAS.COM
9
INS INSIGHTS IAS PRELIMS TEST SERIES - 2017

The doctrine of paramountcy was applicable throughout India


by virtue of which all Indian residents were the subjects of the
British crown.
Q Source: TN Textbook: 12th Standard: Modern History

44. LIMBS Portal has been setup by the Government to

5
WWW.INSIGHTSIAS.COM
0
INS INSIGHTS IAS PRELIMS TEST SERIES - 2017

a) Create a national electronic database of differently abled


people
b) Properly monitor pending court cases of the entire
Government of India
c) Register intellectual property rights online
d) Enable robotics societies in India to collaborate more
effectively
Solution: b)
Learning: Web portal named Legal Information and
Management Based System (LIMBS) has been introduced for
centrally monitoring cases of GoI pending in various courts and
Tribunals.
All the data pertaining to court cases of the Government shall
be available at one place.
The data shall be used for policy planning purposes.
Q Source:
http://pib.nic.in/newsite/PrintRelease.aspx?relid=145669
http://lawmin.nic.in/la/2year-achi.pdf

45. Kalamezhuthu pattu tradition is performed


a) As part of the rituals to worship and propitiate gods in
temples of Kerala
b) On Palm leaf manuscripts to ordain new spiritual
lineages
c) In sacred groves to sensitize the new generation of the
importance of ecology in Western Ghats

5
WWW.INSIGHTSIAS.COM
1
INS INSIGHTS IAS PRELIMS TEST SERIES - 2017

d) By Buddhist monks in South India to initiate new


students
Solution: a)
Learning: The Kalamezhuthu is a forty-day ritualistic festival
beginning with the first of Vrischikam (Scorpio) in most
Bhagavathy temples in Kerala.
This ritualistic art is a common feature of temples as well as
noble households.
The kalam is a unique drawing. The figures drawn usually have
an expression of anger, and other emotions. Kalamezhuthu
artists are generally members of communities like the Kurups,
Theyyampadi Nambiars, Theeyadi Nambiars and Theeyadi
Unnis.
Q Source:
http://www.archive.india.gov.in/knowindia/culture_heritage.p
hp?id=99

46. Consider the following about the re-established


Nalanda University.
1. It was established by an Act of the Parliament.
2. It has been declared as an institution of National
Importance.
3. It is being entirely funded by the World Bank and
donations from foreign governments.
4. Its modern campus is being built replacing the ruins of
the older Nalanda University.
Select the correct answer using the codes below.

5
WWW.INSIGHTSIAS.COM
2
INS INSIGHTS IAS PRELIMS TEST SERIES - 2017

a) 1, 2 and 3 only
b) 2, 3 and 4 only
c) 1 and 2 only
d) 3 and 4 only
Solution: c)
Justification: Statement 1 and 2: Nalanda University was
established by an act of Parliament in 2010 and is designated as
an institution of national importance. Nalanda is a standalone
international university unlike any other established in the
country and is supported by several East Asia Summit
countries.
The President of India is the Visitor of Nalanda University
Statement 3: Despite the international dimension of the project
being repeatedly stressed, the total amount received from the
participating countries in the East Asia summit, including
Australia, China, Thailand, Laos and Indonesia, is paltry at less
than 2 million dollars.
The institute is being financially supported by the Government
of India as well.
Statement 4: The ruins are a national and international heritage
(UNESCO) and cannot be replaced. The new campus is being
built at some distance from the Nalanda ruins.
Q Source:
http://indianexpress.com/article/education/nalanda-
univesity-bihar-education-to-inspire-students-to-fulfill-
societys-expectations-vijay-pandurang-bhatkar-vc-4549745/

5
WWW.INSIGHTSIAS.COM
3
INS INSIGHTS IAS PRELIMS TEST SERIES - 2017

47. Bio-fertilizers increase crop yield on farm lands


because
1. They add a diverse base of nutrients to the soil.
2. They can symbiotically associate with plants improving
their nutrient uptake.
Which of the above is/are correct?
a) 1 only
b) 2 only
c) Both 1 and 2
d) None
Solution: b)
Justification: Statement 1: They dont add any new nutrient
to the soil. These are means of fixing the nutrient availability in
the soil.
Statement 2: Since a bio-fertilizer is technically living, it can
symbiotically associate with plant roots. Involved
microorganisms could readily and safely convert complex
organic material into simple compounds, so that they are easily
taken up by the plants.
It can also provide protection against drought and some soil-
borne diseases.
Q Source:
http://agritech.tnau.ac.in/farm_enterprises/Farm%20enterpri
ses_%20biofertilizer.html : Basic concepts Ecology and
Agriculture

5
WWW.INSIGHTSIAS.COM
4
INS INSIGHTS IAS PRELIMS TEST SERIES - 2017

48. The travel time between which one of the following


cities and Port Blair would be the least by flight?
a) Bangkok
b) Jakarta
c) Manila
d) Flight time is the same in all the above cases.
Solution: a)
Justification: Manila is in Phillipines which is farther away
from the two options graphically illustrated below.

5
WWW.INSIGHTSIAS.COM
5
INS INSIGHTS IAS PRELIMS TEST SERIES - 2017

Q Source: Map based questions

49. Consider the following with reference to Jainism.


1. Jainism spread in South India under the leadership of
Sthalabahu.
2. Chaukyas and Rashtrakutas restricted the Jainas from
expanding their social base in South India, especially if
it included untouchables.
5
WWW.INSIGHTSIAS.COM
6
INS INSIGHTS IAS PRELIMS TEST SERIES - 2017

3. The early Jainas adopted Sanskrit language as their


main language for promoting Jainism in North and
South India.
Select the correct answer using the codes below.
a) 1 and 3 only
b) 2 only
c) 2 and 3 only
d) None of the above
Solution: d)
Justification: Statement 2: In the South, Jainism was
patronised by royal dynasties such as - The (Gangas,
Kadambas, Chalukyas and Rashtrakutas).
Statement 1: One of the important causes of the spread of
Jainism in south India is said to be the great famine that
occurred in Magadha 200 years after the death of Mahavira.

This famine lasted for twelve years and in order to protect


themselves many a Jainas went to the south under the
leadership of Bhadrababu, even Chandragupta Maurya
accompanied him.
But, the rest of them stayed back in Magadha under the
leadership of Sthalabahu. These emigrant Jainas
spread Jainism in south India.
Statement 3: Jainas discarded Sanskrit and adopted Prakrit
language as their main language for promoting it.

Mahavira himself preached in Ardha-Magadhi. The


adoption of Prakrit by the Jainas helped the growth of this
language and its literature.
The Jainas composed the earliest important works in
Apabhramsa and prepared its first grammar.
5
WWW.INSIGHTSIAS.COM
7
INS INSIGHTS IAS PRELIMS TEST SERIES - 2017

Q Source: Improvisation: Past year UPSC papers

50. Which of these states in India is the first to introduce


minimum wages, recently, for part time workers?
a) Maharashtra
b) Assam
c) Bihar
d) Rajasthan
Solution: d)
Learning: Rajasthan has become the first state in the country
to introduce minimum wages for part time workers.
In this regard, the Labour Department has issued a notification
with which part time workers in the state now come under the
Minimum Wages Act 1948.
As per the new rule, it is now mandatory for employer to pay
50% of the prescribed a day minimum wage to a person who
works for less than four hours in a day.
Besides, in another decision state government also has decided
to increase the minimum wages in all categories of scheduled
employments by 104 Rupees per month.
Q Source: http://indianexpress.com/videos/news-
video/rajasthan-govt-fixes-minimum-wages-of-part-time-
workers/

5
WWW.INSIGHTSIAS.COM
8
INS INSIGHTS IAS PRELIMS TEST SERIES - 2017

51. Match the following space probes to their respective


planets/bodies.
1. Curiosity Rover A. Jupiter
2. New Horizons B. Mars
3. Cassini Huygens C. Saturn
4. Juno D. Pluto
Select the correct answer using the codes below.
a) 1-B, 2-D, 3-C, 4-A
b) 1-C, 2-B, 3-A, 4-D
c) 1-D, 2-C, 3-B, 4-A
d) 1-A, 2-b, 3-C, 4-D
Solution: a)
Justification: Statement 1: Curiosity is a car-sized robotic
rover exploring Gale Crater on Mars as part of NASA's Mars
Science Laboratory mission.
Statement 2: New Horizons is an interplanetary space probe.
The primary mission is to perform a flyby study of the Pluto
system. The secondary mission to fly by and study one or more
other Kuiper belt objects (KBOs).
Statement 3: CassiniHuygens is an unmanned spacecraft sent
to the planet Saturn. It is a Flagship-class robotic spacecraft.
Cassini is the fourth space probe to visit Saturn and the first to
enter orbit,
Statement 4: Juno is a NASA space probe orbiting the planet
Jupiter. We have covered it in a test earlier.

5
WWW.INSIGHTSIAS.COM
9
INS INSIGHTS IAS PRELIMS TEST SERIES - 2017

Q Source: Often in news

52. Consider the following about National Productivity


Council (NPC).
1. It is an autonomous organization under the
administrative control of the Department of Rural
Development.
2. It is a non-profit organization with representation from
the government, employers and workers organizations.
Which of the above is/are correct?
a) 1 only
b) 2 only
c) Both 1 and 2
d) None
Solution: b)
Justification: NPC is a national level autonomous
organization under the administrative control of the
Department of Industrial Policy and Promotion to promote
productivity culture in India.

Established as a registered society in 1958 by the


Government of India, it is a tri-partite non-profit
organization with equal representation from the
government, employers and workers organizations, apart
from technical and professional institutions including
members from local productivity councils and chamber of
commerce on its Governing Body.

6
WWW.INSIGHTSIAS.COM
0
INS INSIGHTS IAS PRELIMS TEST SERIES - 2017

Besides providing training, consultancy and undertaking


research in the area of productivity, NPC also implements
the productivity promotion schemes of the Government
and carry out the programmes of the Tokyo based Asian
Productivity Organization (APO), an inter-governmental
body of which the Government of India is a founder
member.
Q Source: DIPP Website

53. With reference to Ancient India, Urvara and Usara


were used to categorize
a) Fertile and barren soils
b) Male-centred and female-centred educational
institutions
c) Taxed and tax-free households
d) Royal and ordinary lineages
Solution: a)
Learning: In India, soil had been classified from the ancient
period itself even though it was not detailed as per the modern
classifications.
In the ancient period, the classification was based on only two
things; whether the soil is fertile (Urvara) or sterile (usara).
In the 16th century A.D., soils were classified on the basis of
their inherent characteristics and external features such as
texture, colour, slope of land and moisture content in the soil.

6
WWW.INSIGHTSIAS.COM
1
INS INSIGHTS IAS PRELIMS TEST SERIES - 2017

Based on texture, main soil types were identified as sandy,


clayey, loamy etc.
Q Source: India Physical Geography: 11th NCERT

54. Water Technology Initiative of the Department of


Science and Technology aims to
1. Promote Research and Development (R&D) activities
aimed at providing safe drinking water at affordable
cost
2. Connecting every household in the country with a piped
water connection from urban local bodies
Which of the above is/are correct?
a) 1 only
b) 2 only
c) Both 1 and 2
d) None
Solution: a)
Justification: This was started in 2007. It aims to achieve
statement 1 through S&T interventions evolved through
indigenous efforts.

Since quality is the main consideration of safe drinking


water, processes which imply nano-material and filtration
technologies have been focused.

6
WWW.INSIGHTSIAS.COM
2
INS INSIGHTS IAS PRELIMS TEST SERIES - 2017

The initiative also includes the pilot testing of credible


number of products and referencing of selected
technologies to the social context of the application region.
It also focuses on capacity building of research
professionals and water managers and evolving
methodology for development of customised solutions
suited to social context.
Q Source: http://www.dst.gov.in News and Events
http://www.dst.gov.in/water-technology-initiative-
programme-wti

55. Mahamaham is a major festival celebrated every 12


years in the South Indian town of
a) Puttaparthi, Andhra Pradesh
b) Kumbakonam, Tamil Nadu
c) Sabarimala, Kerala
d) Tirumala, Andhra Pradesh
Solution: b)
Learning: Hindus consider taking a holy dip at the
Mahamaham tank on the day of Mahamaham as sacred.
Chariot festivals are conducted in the Vishnu temples
alongside.
Astronomically, maham or magha is a nakshatra (constellation)
in Leo sign (Simha Rasi). Since Sun takes a year to go round (as
viewed from earth, as if earth is stationary), this festival is
celebrated in the month when full moon occurs as moon is

6
WWW.INSIGHTSIAS.COM
3
INS INSIGHTS IAS PRELIMS TEST SERIES - 2017

passing Magha nakshatra (Leo sign) and Sun is on the other


end in the opposite Acquarius sign (Kumnha Rasi).
Q Source:
http://www.thehindu.com/news/cities/Tiruchirapalli/sea-of-
pilgrims-at-kumbakonam/article17449258.ece

56. Consider the following about a bacteriophage.


1. It is a bacterium which infects and replicates within
fungi and living plants.
2. It is composed of proteins that encapsulate a DNA or
RNA genome.
3. It is being used as an anti-biotic in some countries.
Select the correct answer using the codes below.
a) 1 only
b) 2 and 3 only
c) 1 and 3 only
d) 2 only
Solution: b)
Justification: Statement 1 and 2: It is a virus that infects and
replicates within a bacterium. Their genomes may encode as
few as four genes, and as many as hundreds of genes.

Phages replicate within the bacterium following the


injection of their genome into its cytoplasm.
Bacteriophages are among the most common and diverse
entities in the biosphere.
6
WWW.INSIGHTSIAS.COM
4
INS INSIGHTS IAS PRELIMS TEST SERIES - 2017

Statement 3: They have been used for long as an alternative to


antibiotics in the former Soviet Union and Central Europe, as
well as in France. They are seen as a possible therapy against
multi-drug-resistant strains of many bacteria
However, there are serious side effects involved that need to be
taken care of.
Q Source:
https://www.sciencedaily.com/releases/2017/03/17030310042
9.htm
http://www.the-
scientist.com/?articles.view/articleNo/47698/title/Phages-
Carry-Antibiotic-Resistance-Genes/
https://www.ncbi.nlm.nih.gov/pubmed/24619620

57. Which of these is/are the key features of Draft Model


Agricultural Produce Market Committee (APMC) Act 2016
released by the Ministry of Agriculture and Farmers
Welfare?
1. It strives to create a barrier free single market for
agricultural products in the country.
2. It bans the participation of private market yards in
agricultural trading operations.
3. It abolishes all forms of market fees on agricultural
produce sold in mandis.
Select the correct answer using the codes below.
a) 1 and 3 only
b) 2 only
c) 1 only
6
WWW.INSIGHTSIAS.COM
5
INS INSIGHTS IAS PRELIMS TEST SERIES - 2017

d) 1, 2 and 3
Solution: c)
Justification: Statement 1: The model Act (which states are
exhorted to adopt) seeks to facilitate free flow of agricultural
produce and direct interface of farmers with exporters or end
users.
A state government/UT administration may declare the whole
state/UT as a single unified market area for the purpose of
regulating agricultural produce.
Statement 2: Private market yards may be set up to facilitate
operations of traders, commission agents, etc. Further, farmer-
consumer market yards may be set up by providing
infrastructure accessible to farmers and consumers directly.
Statement 3: The Market Committee shall levy market fee on
agricultural produce from a buyer only once (single point levy),
whether brought from outside or within the state/UT.
Learning: A state may set up an electronic trading platform,
which shall provide infrastructure and services for trading in
agricultural produce. A person may obtain license to establish
and run an e-trading platform. Further, e-trading platforms
maybe integrated with private market yards and sub-yards.
Under a unified National Agricultural Market, e-platforms shall
be interoperable.
Q Source: http://agricoop.nic.in/recentinitiatives/draft-
model-apmc-acts-2016

6
WWW.INSIGHTSIAS.COM
6
INS INSIGHTS IAS PRELIMS TEST SERIES - 2017

58. Which of the following could NOT have been a


demand of the moderate faction of the Indian National
Congress during the freedom struggle?
a) Reduction of land revenue and protection of peasants
from unjust landlords
b) Boycott of legislative councils and government
institutions
c) Freedom of speech and expression and freedom to form
associations
d) More powers for the local bodies
Solution: b)
Justification & Learning: The moderates believed in the
politics of coordination and cooperation, where the British
could be peaefullly requested to submit to their demand,
something the extremists called the politics of mendicancy.
Some other demands of moderates were:

Greater opportunities for Indians in higher posts by


holding the ICS examination simultaneously in England
and in India.
Separation of the judiciary from the executive.
Abolition of salt tax and sugar duty.
Reduction of spending on army
Q Source: TN Textbook: 12th Standard History

59. The soil of tropical rainforests is leached mainly due


to
1. High amount of organic matter accumulated in the soil
2. High precipitation and temperature that facilitates
washing away of soil
6
WWW.INSIGHTSIAS.COM
7
INS INSIGHTS IAS PRELIMS TEST SERIES - 2017

Which of the above is/are correct?


a) 1 only
b) 2 only
c) Both 1 and 2
d) None
Solution: b)
Justification: Tropical rainforests can be characterized as hot
and highly humid. Mean monthly temperatures exceed 18 C
during all months of the year and annual rainfall lies between
175-300 cm.
This high level of precipitation often results in poor soils due to
leaching of soluble nutrients in the ground.
Moreover, high temperature also promotes activity of microbes
that work continuously on the soil.
Q Source: 8th NCERT: Geography

60. Which of these is/are the responsibilities of the


Petroleum and Explosives Safety Organisation (PESO)?
1. To fix and revise the import and export duties of
petroleum related products in India
2. To evaluate new technologies for finding applications in
hazardous area for adoption in indigenous conditions
Which of the above is/are correct?
a) 1 only
b) 2 only
6
WWW.INSIGHTSIAS.COM
8
INS INSIGHTS IAS PRELIMS TEST SERIES - 2017

c) Both 1 and 2
d) None
Solution: b)
Justification: It is the nodal agency to look after safety
requirements of the Explosives and Petroleum Sectors.
Statement 1: To administer the responsibilities delegated under
the Explosives Act 1884 and Petroleum Act 1934 and the rules
made there under related to manufacture, import, export,
transport, possession, sale and use of Explosives, Petroleum
products and Compressed gases is its responsibility apart from
providing related technical guidance.
Statement 2: It must ensure public safety in the areas of
manufacture, transport, storage, handling, etc. of Explosives,
Petroleum, Carbide of Calcium, Inflammable substances and
Compressed Gases.
Evaluation of new technologies in hazardous within the purview
of Explosives Act 1884 and Petroleum Act 1934 also comes
under its ambit.
You can read more here
http://peso.gov.in/Roles_Respons.aspx
Q Source:
http://dipp.nic.in/English/AboutUs/statutory_bodies.aspx

61. Consider the following statements about popular


socio-religious movements in various regions of India.

6
WWW.INSIGHTSIAS.COM
9
INS INSIGHTS IAS PRELIMS TEST SERIES - 2017

1. The Aligarh Movement focussed on the social and


educational advancement of the Muslims in India.
2. The Nirankari Movement insisted on the worship of
formless God.
3. The Namdhari Movement followers wore white clothes
and practiced vegetarianism.
Select the correct answer using the codes below.
a) 2 only
b) 1 and 3 only
c) 2 and 3 only
d) 1, 2 and 3 only
Solution: d)
Justification: Statement 1: The Aligarh Movement was
started by Sir Syed Ahmad Khan (1817-98) for the social and
educational advancement of the Muslims in India. He fought
against the medieval backwardness and advocated a rational
approach towards religion.
Statement 2 and 3: Punjab was also influenced by reform
movements. Baba Dayal Das founded the Nirankari Movement.
He insisted the worship of God as nirankar (formless). The
Namdhari Movement was founded by Baba Ram Singh. His
followers wore white clothes and gave up meat eating.
The Singh Sabhas started in Lahore and Amritsar in 1870 were
aimed at reforming the Sikh society.
Q Source: TN Textbook: 12th Standard: Modern History

7
WWW.INSIGHTSIAS.COM
0
INS INSIGHTS IAS PRELIMS TEST SERIES - 2017

62. Which of the following rivers is most likely to make


an estuary?
a) Tapi
b) Mahanadi
c) Krishna
d) Brahmaputra
Solution: a)
Learning: In the above options, Tapi is the only west flowing
river (of the Peninsular India).Rest are east flowing rivers
which make delta.
West flowing rivers flow through shorter and harder terrain and
thus generate fewer sediments as compared to their east
flowing counterparts.
Moreover, the gradient is steep and their flow speed is much
faster which hinders delta formation in the Arabian sea. Hence,
they form estuaries, for e.g. Narmada.
Q Source: ICSE Geography Textbook: Class IX

63. In colonial India, which among the following were


major centres of textile production?
1. Chanderi
2. Dacca
3. Arni
4. Banaras
Select the correct answer using the codes below.
a) 2 and 4 only
7
WWW.INSIGHTSIAS.COM
1
INS INSIGHTS IAS PRELIMS TEST SERIES - 2017

b) 1 and 3 only
c) 1, 2 and 3 only
d) 1, 2, 3 and 4
Solution: d)
Justification: Indias cotton, silk and woolen products were
sought after all over the world. Particularly, the muslin of
Dacca, carpets of Lahore, shawls of Kashmir, and the
embroidery works of Banaras were very famous.

Apart from Dacca, which was highly famous for its


muslins, the other important centres of textile production
were Krishnanagar, Chanderi, Arni and Banaras. Dhotis
and dupattas of Ahmedabad, Chikan of Lucknow, and silk
borders of Nagpur had earned a worldwide fame.
For their silk products some small towns of Bengal
besides, Malda and Murshidabad were very famous.
Similarly, Kashmir, Punjab and western Rajasthan were
famous for their woolen garments.
Q Source: TN Textbook: 12th Standard: Modern History

64. On what basis are firms categorized in Micro, Small


and Medium Enterprise (MSME) categories in India?
a) Investment in plant and machinery
b) Pan-India versus local presence
c) Compliance to local and state laws
d) Employment Generation potential
Solution: a)

7
WWW.INSIGHTSIAS.COM
2
INS INSIGHTS IAS PRELIMS TEST SERIES - 2017

Learning: Enterprises are classified into two categories -


manufacturing enterprises and service enterprises.
The limit for investment in plant and machinery / equipment
for manufacturing / service enterprises is as under:

You need not remember these figures, but only get a rough idea
of the scale of investment.
Q Source: 11th NCERT: Indian Economic Development

65. Consider the following about Pattachitra paintings.


1. They are cloth based scroll paintings.
2. They are practiced in Odisha.
3. They are completely secular or non-religious in nature.
Select the correct answer using the codes below.
a) 2 only
b) 2 and 3 only
c) 1 and 2 only
d) 1 and 3 only
7
WWW.INSIGHTSIAS.COM
3
INS INSIGHTS IAS PRELIMS TEST SERIES - 2017

Solution: c)
Justification: It is a general term for traditional, cloth-based
scroll painting based in Odisha.

Most of these paintings depict stories of Hindu deities. So,


3 is wrong.
These paintings are based on Hindu mythology and
specially inspired by Jagannath and Vaishnava sect.
All colours used in the Paintings are natural. The
'pattachitra' resemble the old murals of Odisha especially
religious centres of Puri, Konark and Bhubaneshwar
region, dating back to the 5th century BC.
Q Source:
http://www.archive.india.gov.in/knowindia/culture_heritage.p
hp?id=99

66. The Indian Technical and Economic Cooperation


(ITEC) Programme assists
a) Indian Universities for Higher education
b) Developing countries both financially and non-
financially
c) Linguistic and religious Minorities all across the World
d) All multi-national corporations operating in India
Solution: b)
Learning: It is a major plank of Indias South-South foreign
policy.

7
WWW.INSIGHTSIAS.COM
4
INS INSIGHTS IAS PRELIMS TEST SERIES - 2017

Instituted in 1964, it is a bilateral programme of assistance of


the Government of India.

Under ITEC and its sister programme SCAAP (Special


Commonwealth African Assistance Programme), 161
countries in Asia, Africa, East Europe, Latin America etc.
are invited to share in the Indian developmental
experience acquired over six decades of India's existence
as a free nation.
As a result of different activities under this programme,
there is now a visible and growing awareness among other
countries about the competence of India as a provider of
technical know-how and expertise as well as training
opportunities, consultancy services and feasibility studies.
These programmes have generated immense goodwill and
substantive cooperation among the developing countries.
Q Source: MEA Website

67. Apart from nucleus, in the cells of living plants,


which of the following organelles contain DNA?
a) Centrioles
b) Golgi apparatus
c) Endoplasmic reticulum
d) Chroloplast
Solution: d)
Learning: Two cell-like organelles contain DNA

Mitochondria, which is found in almost all eukaryotes.


Mitochondria carries out cellular respiration. This process

7
WWW.INSIGHTSIAS.COM
5
INS INSIGHTS IAS PRELIMS TEST SERIES - 2017

uses the energy in sugars, fats and proteins to make ATP.


ATP is used for cellular work.
Chloroplasts, which is found only in plants and algae.
Chloroplasts convert solar energy to chemical energy in
sugars. It possess circular DNA and divides by fission,
similar to bacteria.
Q Source: Improvisation: Past year UPSC papers

68. Equalisation Levy, recently seen in news, will be


applied on
a) Online advertisements
b) Emission of Greenhouse gases (GHGs)
c) International medical tourists
d) Land under Public Private Partnership (PPP) mode
Solution: a)
Learning: Also known as Google Tax, it will apply to payments
for online advertisements made by Indian business entities to
non-residents (such as Google, Yahoo, Twitter, Facebook)
where the aggregate payment in a financial year to a non-
resident exceeds Rs one lakh.
Only Business to Business (B2B) transactions attract this levy.
India became the first country to impose such an equalisation
levy.
You can read more about it at Q Source.
Q Source:
http://economictimes.indiatimes.com/news/economy/policy/g

7
WWW.INSIGHTSIAS.COM
6
INS INSIGHTS IAS PRELIMS TEST SERIES - 2017

oogle-tax-could-soon-make-even-downloading-apps-on-your-
smartphone-a-costly-affair/articleshow/57539781.cms

69. Light pollution can have which of the following


effects on the ecology and biodiversity?
1. It is known to disturb the reproductive cycles of some
animals.
2. It adversely affects the migration of birds that navigate
using the stars.
3. It disturbs circadian rhythms in humans affecting the
sleep pattern.
Select the correct answer using the codes below.
a) 1 only
b) 2 and 3 only
c) 1 and 3 only
d) 1, 2 and 3
Solution: d)
Justification: Light pollution is excessive brightness that
causes visual discomfort.

Skyglow is the brightening of the night sky over inhabited


areas. It is caused by cluttered, bright, excessive and
confusing groupings of light sources.
It affects the reproductive cycles of those animals that
depend on sensing light movements or seasonal
movements to reproduce. Since birds migrate using star

7
WWW.INSIGHTSIAS.COM
7
INS INSIGHTS IAS PRELIMS TEST SERIES - 2017

light, it can confuse them, and even disorient night-flying


insects.
Excessive blue light emitted from LEDs directly affects
sleep pattern in Human by suppressing the production of
the hormone melatonin, which mediates the sleep-wake
cycle in humans.
Q Source: http://www.hindustantimes.com/health-and-
fitness/this-earth-hour-here-s-another-reason-to-turn-the-
switch-off-light-pollution/story-
C5M6UhuDihSB5aSqNvECnI.html

70. The Social and Infrastructure Development Fund


(SIDF) has been created for which of the following regions
in India?
a) Regions of high communicable disease incidence
b) North Eastern Region (NER)
c) All mining affected regions of India
d) Jammu and Kashmir (J&K)
Solution: b)
Learning: It was created in the Public Account for North
Eastern Region (NER), especially for Arunachal Pradesh and
other border areas facing special problems that cannot be
tackled through normal schemes.
The initiatives included Upgradation of 1396 ITIs, training of
Farmers, Employment for Physically Challenged, Means cum-
Merit Scholarships, Ground Water Recharge, Social Security
through provision of death and disability insurance cover etc.
7
WWW.INSIGHTSIAS.COM
8
INS INSIGHTS IAS PRELIMS TEST SERIES - 2017

It was initiated in 2008-09 budget and continued in the present


budget.
Q Source:
http://pib.nic.in/newsite/PrintRelease.aspx?relid=160152

http://www.archive.india.gov.in/knowindia/culture_heritage.p
hp?id=16

71. Which of these statements about the Prarthana


Samaj is correct?
a) It was founded after the Non-cooperation movement
was winded up.
b) It was an off-shoot of Brahmo Samaj.
c) It was a spiritual and educational movement focussing
on reviving University education.
d) It was established by Mahadev Govind Ranade.
Solution: b)
Justification & Learning: It was founded in 1867 in Bombay
by Dr. Atmaram Pandurang as an off-shoot of Brahmo Samaj.
It was a reform movement within Hinduism and concentrated
on social reforms like inter-dining, inter-marriage, widow
remarriage and uplift of women and depressed classes.
Justice M.G. Ranade and R.G. Bhandarkar joined it in 1870 and
infused new strength to it. Justice Ranade promoted the Deccan
Education Society.
Q Source: TN Textbook: 12th Standard: Modern History

7
WWW.INSIGHTSIAS.COM
9
INS INSIGHTS IAS PRELIMS TEST SERIES - 2017

72. Which of these Indian cities have been recently


included in the Tentative list of UNESCO World Heritage
Sites?
1. Delhi
2. Jaipur
3. Ahmadabad
4. Kurnool
Select the correct answer using the codes below.
a) 1, 2 and 3 only
b) 3 and 4 only
c) 1 and 2 only
d) 2, 3 and 4 only
Solution: a)
Justification: Five Indian cities have been included in the
Tentative list of UNESCO World Heritage Sites, namely,
Historic City of Ahmadabad, Delhi A Heritage City, Jaipur
City, Rajasthan, Ekamra Kshetra- The Temple City,
Bhubaneswar and The Victorian Art Deco Ensemble of
Mumbai.
Tentative list nominations are generally considered in future to
be awarded as UNESCO World Heritage sites.
Some of the important Criteria for consideration in the list are:

To represent a masterpiece of human creative genius;


To exhibit an important interchange of human values
To bear a unique or at least exceptional testimony to a
cultural tradition or to a civilization which is living or
which has disappeared;
8
WWW.INSIGHTSIAS.COM
0
INS INSIGHTS IAS PRELIMS TEST SERIES - 2017

To be an outstanding example of a type of building,


architectural or technological ensemble or landscape
Q Source: UNESCO World Heritage List: Tentative

73. Every Panchayat, unless sooner dissolved under any


law for the time being in force, shall continue for five years
from the date appointed for its
a) Taking of oath of office by the elected members
b) First audit by Gram Sabha
c) Declaration of the election results
d) Its first meeting
Solution: d)
Learning: Panchayat is an institution of self-government
constituted under article 243B, for the rural areas.
Duration is five years from the date of its first meeting.
Moreover, an election to constitute a Panchayat shall be
completed before the expiry of its duration or before six months
from the date of its dissolution.
A Panchayat constituted upon the dissolution of a Panchayat
before the expiration of its duration shall continue only for the
remainder of the period for which the dissolved Panchayat
would have continued.
Q Source: Chapter on PRIs: Indian Polity: M Laxmikanth

8
WWW.INSIGHTSIAS.COM
1
INS INSIGHTS IAS PRELIMS TEST SERIES - 2017

74. The Palayakkarar system had evolved with the


extension of Vijayanagar rule into Tamil Nadu. Each
Palayakkarar was
a) The holder of a territory granted to him in return for
military service and tribute
b) A sole judge for all civil and criminal cases in his cause
in a defined territory
c) The head of a group of mandalas that were sub-units of
villages in the kingdom
d) A religious leader who claimed tax exemption on
account of their duties to God
Solution: a)
Learning: The question is with reference to the revolt of the
Palayakkarars (Poligars) against the East India Company.

The Palayakkarars were often interested in increasing


their own powers. With their numerical strength,
extensive resources, local influence and independent
attitude, the Palayakkarars came to constitute a powerful
force in the political system of south India.
They regarded themselves as independent, sovereign
authorities within their respective Palayams, arguing that
their lands had been handed down to them across a span
of generations.
Such claims were brushed aside by the East India
Company, which eventally led to the uprising.
Q Source: TN Textbook: 12th Standard: Modern History

8
WWW.INSIGHTSIAS.COM
2
INS INSIGHTS IAS PRELIMS TEST SERIES - 2017

75. Consider the following statements with reference to


the Kyoto Protocol.
1. The Clean Development Mechanism enables developed
countries to invest in Clean projects in developing
countries to gain emissions credits.
2. Developing countries like India have no mandatory
mitigation obligations or targets under the Kyoto
Protocol.
3. The Government of India has decided to reject the
Second Commitment Period of the Kyoto Protocol.
Select the correct answer using the codes below.
a) 2 only
b) 1 and 2 only
c) 1 and 3 only
d) 1, 2 and 3
Solution: b)
Justification: Statement 1: Developed countries get Certified
Emission Reduction-CER. Indian Industry has benefited from
trading in CERs in the international market. Of late, due to lack
of demand internationally, prices of CERs have been very low
leading to low activities under this mechanism.
Statement 3: India has ratified the 2013- 2020 commitment
period of the Protocol to continue its effort towards voluntary
GHG emission reduction.
Q Source: Major international Environmental protocols

8
WWW.INSIGHTSIAS.COM
3
INS INSIGHTS IAS PRELIMS TEST SERIES - 2017

76. Consider the following statements.


Assertion (A): ISRO has decided to remove all lithium ion
batteries from its satellite and launch vehicle applications.
Reason (R): Lithium-ion batteries can pose unique safety
hazards since they contain a flammable electrolyte.
In the context of the above, which of these is correct?
a) A is correct, and R is an appropriate explanation of A.
b) A is correct, but R is not an appropriate explanation of
A.
c) A is correct, but R is incorrect.
d) A is incorrect, but R is correct.
Solution: d)
Justification: ISRO has been successfully using lithium ion
battery for satellite and launch vehicle applications. But,
recently ISRO in collaboration with Automotive Research
Association of India (ARAI) has developed Li-ion batteries for
use in electric vehicles.
This is a major breakthrough since the country imports 100% of
the lithium battery requirements, and so far, there is no
manufacturing in the country. ISROs collaboration with ARAI
will help indigenize this technology.
Q Source:
http://www.financialexpress.com/industry/technology/homeg
rown-lithium-battery-to-power-indias-electric-
vehicles/514977/

8
WWW.INSIGHTSIAS.COM
4
INS INSIGHTS IAS PRELIMS TEST SERIES - 2017

77. India INX, recently seen in news, is Indias first


a) International exchange centre
b) Data Centre hosted only by supercomputers
c) Multi-cultural handloom and powerloom government
outlet
d) Single window mechanism for providing walk-in
employment to youth
Solution: a)
Learning: It was established at the International Financial
Service Centre (IFSC) of GIFT (Gujarat International Financial
Tech) City Gandhinagar.

India INX is a wholly-owned subsidiary of the Bombay


Stock Exchange (BSE). It will enable Indian firms to
compete on equal footing with offshore firms.
It will facilitate international investors and NRIs to trade
from anywhere in the world.
It will provide benefits in terms of waiver of security
transaction tax, commodity transaction tax, dividend
distribution tax, long term capital gain tax and income tax.
You can find valuable information at question source.
Q Source: http://www.business-
standard.com/article/markets/bse-s-india-inx-10-things-to-
know-about-our-1st-international-exchange-
117011000301_1.html

8
WWW.INSIGHTSIAS.COM
5
INS INSIGHTS IAS PRELIMS TEST SERIES - 2017

78. How the phenomenon of Weathering is important for


us?
1. It aids in the formation of soil by eroding rocks and
hard surfaces.
2. It helps in the enrichment of valuable minerals on earth
for their commercial extraction to be made possible.
3. It is responsible for shaping and denuding large
landforms on earth.
Select the correct answer using the codes below.
a) 1 and 2 only
b) 2 only
c) 1 and 3 only
d) 1, 2 and 3
Solution: d)
Justification: Statement 1: Weathering processes are
responsible for breaking down the rocks into smaller fragments
and preparing the way for formation of not only regolith and
soils, but also erosion and mass movements.
Biomes and biodiversity is basically a result of forests
(vegetation) and forests depend upon the depth of weathering
mantles.
Erosion cannot be significant if the rocks are not weathered.
Statement 2: Without weathering taking place, the
concentration of the same valuable material may not be
sufficient and economically viable to exploit, process and refine
since they would be scattered across a large area.

8
WWW.INSIGHTSIAS.COM
6
INS INSIGHTS IAS PRELIMS TEST SERIES - 2017

Statement 3: Weathering aids mass wasting, erosion and


reduction of relief and changes in landforms are a consequence
of erosion.
Q Source: Fundamental of Physical Geography: 11 th NCERT

79. On the north, north-west and north-eastern sides,


India is largely bounded by
a) Block mountains
b) Fold mountains
c) Volcanic mountains
d) Plateau Dome Mountains
Solution: b)
Learning: The Himalayas stretches across around 2,900
kilometres of India, Pakistan, China, and Nepal borders. It
covers most of the Northern and North-east boundaries of
India.
Fold Mountains are created where two or more of Earths
tectonic plates are pushed together, often at regions known as
convergent plate boundaries and continental collision zones.
Q Source: 6th NCERT Geography

80. FAME India Scheme adopted by the Government of


India concerns
8
WWW.INSIGHTSIAS.COM
7
INS INSIGHTS IAS PRELIMS TEST SERIES - 2017

a) Spreading literary and research works of famous


personalities in India
b) Promoting hybrid and electric mobility vehicles
c) Conservation and display of ancient Indian heritage
abroad
d) Effective ban and regulation of ponzi and related money
making schemes
Solution: b)
Learning: The objective of the scheme is to support
hybrid/electric vehicles market development and
Manufacturing eco-system. The scheme has 4 focus areas i.e.
Technology development, Demand Creation, Pilot Projects and
Charging Infrastructure
Under Faster Adoption and Manufacturing of (Hybrid and)
Electric Vehicles (FAME-India) Scheme of the Government,
demand incentives are being extended to all vehicle segments,
including buses and light commercial vehicles.
You can read about the quantum and other details of the
scheme at the Q Source.
Q Source:
http://pib.nic.in/newsite/mbErel.aspx?relid=154119

81. Arthrospira is cultivated worldwide and often used as


a dietary supplement and whole food. This popularity of
arthrospira is because
1. It is rich in proteins harbouring essential amino acids.

8
WWW.INSIGHTSIAS.COM
8
INS INSIGHTS IAS PRELIMS TEST SERIES - 2017

2. It can be produced in local ponds, not requiring large


tracts of land.
Which of the above is/are correct?
a) 1 only
b) 2 only
c) Both 1 and 2
d) None
Solution: c)
Justification: Statement 1: Arthrospira is a genus of free-
floating filamentous cyanobacteria.
Arthrospira is very rich in proteins, contents range more than
half of its dry weight. Its protein harbours all essential amino
acids as well as high amounts of polyunsaturated fatty acids
(PUFAs), vitamins, Minerals and Photosynthetic pigments.
Statement 2: Species of the genus Arthrospira have been
isolated from alkaline brackish and saline waters in tropical and
subtropical regions.
Most cultivated spirulina is produced in open-channel raceway
ponds, with paddle-wheels used to agitate the water.
The largest commercial producers of spirulina are located in the
United States, Thailand, India, Taiwan, China, Pakistan, Burma
(a.k.a. Myanmar), Greece and Chile
Q Source: Improvisation: Past year UPSC papers

8
WWW.INSIGHTSIAS.COM
9
INS INSIGHTS IAS PRELIMS TEST SERIES - 2017

82. Consider the following about the duties and


responsibilities of the Comptroller and Auditor General of
India (CAG).
1. He audits the accounts related to all expenditure from
the Consolidated Fund of India and Consolidated fund
of each state.
2. He is an ex-officio member of the Public Accounts
Committee guiding its conduct of financial affairs.
3. He is an advisory member of the Department of
Economic Affairs suggesting economy cuts in the design
of annual budget.
Select the correct answer using the codes below.
a) 1 only
b) 3 only
c) 2 and 3 only
d) 1 and 2 only
Solution: a)
Justification: Statement 2: He is called by the PAC to help it
examine the government expenditure in the last financial year.
CAG is not a member of the PAC.
Learning: CAGs (Duties, Powers and Conditions of Service)
act, 1971 , the Constitution and some rules made there in
regulate CAGs duties.

He also audits the accounts of consolidated fund of each


union territory having a Legislative Assembly.
He audits all expenditure from the Contingency Fund of
India and the Public Account of India as well as the
contingency fund of each state and the public account of
each state.

9
WWW.INSIGHTSIAS.COM
0
INS INSIGHTS IAS PRELIMS TEST SERIES - 2017

He audits all trading, manufacturing, profit and loss


accounts, balance sheets and other subsidiary accounts
kept by any department of the Central Government and
state governments.
He audits the receipts and expenditure of the Centre and
each state to satisfy himself that the rules and procedures
in that behalf are designed to secure an effective check on
the assessment, collection and proper allocation of
revenue.
He also audits government companies.
Q Source: Chapter on CAG: Indian Polity: M Laxmikanth

83. Consider the following about Quick Response code


(QR code).
1. Every such code is created using black and white circles.
2. It is capable of recording more data than a traditional
bar code.
3. The Union Government has launched Bharat QR code
to enable digital payments without using card swiping
machines.
Select the correct answer using the codes below.
a) 1 and 2 only
b) 2 and 3 only
c) 3 only
d) 1 and 3 only
Solution: b)

9
WWW.INSIGHTSIAS.COM
1
INS INSIGHTS IAS PRELIMS TEST SERIES - 2017

Justification: Statement 1: QR code (Quick Response code) is


a two-dimensional (matrix) machine- readable bar code made
up of black and white square. This code can be read by the
camera of a smartphone.
Statement 2: It is used for storing URLs or other information
that link directly to text, emails websites phone numbers. It is
capable of 360 degrees (omni-directional), high speed reading.

QR Code can store up to 7089 digits as compared to


conventional bar codes which can store max 20 digits. It
encodes same amount of data in one-tenth the space of a
traditional bar code.
It carries information both horizontally and vertically. It
has error correction capability and data stored in it can be
restored even if it is partially damaged or dirty.
Statement 3: Bharat QR code is worlds first interoperable
payment acceptance solution launched by Indian Government
to move towards less-cash economy.
It will make payments seamless for buyers as they just have to
"scan to pay" for transactions instead of swiping their
credit/debit cards.
Q Source:
http://economictimes.indiatimes.com/industry/banking/finan
ce/bharat-qr-code-launched-to-push-less-cash-
economy/articleshow/57256004.cms

9
WWW.INSIGHTSIAS.COM
2
INS INSIGHTS IAS PRELIMS TEST SERIES - 2017

84. The planet Neptune appears blue in colour mainly


due to its
a) Icy debris
b) Gas bubbles of methane
c) Distance from the Sun
d) Rate of rotation on its own axis
Solution: b)
Learning: Neptune's atmosphere is made up of hydrogen,
helium and methane.
The methane in Neptune's upper atmosphere absorbs the red
light from the sun but reflects the blue light from the Sun back
into space.
This is why Neptune appears blue.
Option D: If a planet rotates too fast on its own axis, it would
most likely appear white in colour, as seen in the Newton Disc
(where rainbow colours when rotated at high rpm will project a
near white gray color).
Q Source: Improvisation: Past year UPSC papers

85. Overlooking the plateau of Malwa to the north and


the valley of the Narmada River to the south which acted
as natural defences for the fort capital of the Paramaras, it
is a ruined city renowned for its fine architecture?
a) Mandu
b) Vaishali
9
WWW.INSIGHTSIAS.COM
3
INS INSIGHTS IAS PRELIMS TEST SERIES - 2017

c) Lothal
d) Somnath
Solution: a)
Justification: Elimination easily gives the right answer.
Malwa plateau is located in Central India, whereas Lothal and
Somnath are located in North-western and western India. So, C
and D are wrong.
Vaishali, in Bihar, does not overlook Narmada to the South, and
hence B is also wrong. Correct answer is A.
Learning: Mandu gained prominence in 10th and 11th century
under the Paramaras. It is perched on the Vindhya Range. As
"Mandapa-Durga", Mandu is mentioned as the royal residence
in the inscriptions of the Paramara kings starting from
Jayavarman II.

Mandu or Mandavgad is a ruined city in the present-day


Mandav area of the Dhar district.
It is located in the Malwa region of western Madhya
Pradesh. This fortress town on a rocky outcrop about
hundred km from Indore is celebrated for its fine
architecture.
Mandu, due to its strategic position and natural defences,
was an important place with a rich and varied history.
It was an important military outpost and its military past
can be gauged by the circuit of the battlemented wall,
which is nearly 37 km and is punctuated by 12 gateways.
Q Source: History and architecture of India: MP Government
Website

9
WWW.INSIGHTSIAS.COM
4
INS INSIGHTS IAS PRELIMS TEST SERIES - 2017

86. Which of the following is the correct decreasing


sequence in terms of Indias contribution to world
production of these commodities?
a) Coal & Lignite> Bauxite> Iron Ore> Petroleum
b) Petroleum > Coal & Lignite> Iron Ore> > Bauxite
c) Iron Ore> Petroleum > Bauxite> Coal & Lignite
d) Bauxite> Iron Ore> Petroleum > Coal & Lignite
Solution: a)
Learning: Please refer to the data below.

9
WWW.INSIGHTSIAS.COM
5
INS INSIGHTS IAS PRELIMS TEST SERIES - 2017

9
WWW.INSIGHTSIAS.COM
6
INS INSIGHTS IAS PRELIMS TEST SERIES - 2017

Q Source: Ministry of Mines Annual Report:


http://mines.nic.in/writereaddata/UploadFile/Annual%20Rep
ort%202014-15635699605386348293.pdf

87. The Ozone Cell of the Ministry of Environment,


Forests and Climate Change (MoEF&CC) renders
necessary services for effective and timely implementation
of the
a) Montreal Protocol
b) Geneva Protocol
c) Basel Convention
d) Chicago Convention
Solution: a)
Learning: The Vienna Convention for the Protection of the
Ozone Layer and the Montreal Protocol on Substances that
Deplete the Ozone Layer are the international treaties for the
protection of the Ozone layer. India became Party to the Vienna
Convention and the Montreal Protocol in 1991-92.
Currently, the Ozone Cell is engaged in phase-out of production
and consumption of next category of chemicals, Hydro
chlorofluorocarbons (HCFCs) with an accelerated phase-out
schedule as per the Montreal Protocol.
Q Source: http://www.moef.nic.in/division/montreal-
protocol-ozone-cell-oc

9
WWW.INSIGHTSIAS.COM
7
INS INSIGHTS IAS PRELIMS TEST SERIES - 2017

88. The Ilbert Bill Controversy is a high watermark in the


history of Indian National Movement. It is related to
a) Abolition of discrimination between natives and
Europeans in Judicial proceedings
b) Dangerous working conditions for workers in factories
located in hinterlands
c) Ban of indigenous press and missionary activities by the
state
d) Derailment of political organizations fighting for Indias
independence due to stringent statutory requirements
Solution: a)
Learning: According to the system of law, a European could
be tried only by a European Judge or a European Magistrate.
The disqualification was unjust and it was sought to cast a
needless discredit and dishonour upon the Indian-born
members of the judiciary.

C.P. Ilbert, Law Member, introduced a bill in 1883 to


abolish this discrimination in judiciary. But Europeans
opposed this Bill strongly.
They also suggested that it was better to end the English
rule in India than to allow the English to be subjected to
the Indian Judges and Magistrates. The bill ended due to
this opposition.
The Ilbert Bill controversy helped the cause of Indian
nationalism. The immediate result of this awakening of
India was the birth of the Indian National Congress in
1885, the very next year of Ripons departure.
Q Source: TN Textbook: 12th Standard: Modern History

9
WWW.INSIGHTSIAS.COM
8
INS INSIGHTS IAS PRELIMS TEST SERIES - 2017

89. Which of the following statements weakens the thesis


that culture strongly shapes the way we see the world?
1. Human tendencies and biases often have a genetic
origin.
2. The modern society places a lot of emphasis on formal
education of natives.
Which of the above is/are correct?
a) 1 only
b) 2 only
c) Both 1 and 2
d) None
Solution: c)
Justification: Statement 1: If our behaviour also has a genetic
origin, then cultural upbringing is likely to have a lesser effect
on the way we behave and see the see the world. It is one of the
ongoing debates, Nature vs Nurture?
Statement 2: Formal education, which is similar almost
everywhere in the world, instills some standardized sets of
values in students irrespective of their culture (for e.g. MBA
schools inculcate business values in their students irrespective
of their backgrounds). This affects the behaviour and attitude of
natives, and thus weakens the assertion in the question
statement.

9
WWW.INSIGHTSIAS.COM
9
INS INSIGHTS IAS PRELIMS TEST SERIES - 2017

Q Source: Improvisation: Recent Speech: Vice-President of


India: PIB Releases

90. Earth Hour is an annual international event


organised by the
a) World Wide Fund for Nature (WWF)
b) United Nations Environment Programme (UNEP)
c) United Nations Framework Convention on Climate
Change (UNFCCC)
d) Conservation International (CI)
Solution: a)
Learning: It is held annually in end of March month to
encourage everyone to turn off their non-essential lights for one
hour, from 8:30 to 9:30 p.m.
The event encourages households and businesses to turn off
their lights and electrical appliances for one hour at the
appointed time to raise awareness about the need to take action
on climate change.
Its goal is to raise awareness for sustainable energy use and
create a more environmentally sustainable lifestyle.
Q Source: WWF Website: https://www.worldwildlife.org/

1
WWW.INSIGHTSIAS.COM 0
0
INS INSIGHTS IAS PRELIMS TEST SERIES - 2017

91. Consider the following about Surrogacy practices in


India.
1. Commissioning surrogacy in India is presently
prohibited for foreigners.
2. Surrogacy Rules, 2015 mandate a standard
methodology and compensation for both public and
private sector surrogacy in India.
Which of the above is/are correct?
a) 1 only
b) 2 only
c) Both 1 and 2
d) None
Solution: a)
Justification: Statement 1: It is prohibited for foreigners,
Persons of Indian Origin (PIOs) and Overseas Citizen of India
(OCI) vide Ministry of Home Affairs Order of 2015.
Statement 2: As of now, there is no order, rule or law that
prescribes a standard surrogacy rate or charge in India.
However, the Surrogacy (Regulation) Bill, 2016, introduced in
the Lok Sabha envisages provisions to govern surrogacy in
India in all its aspects.
It will also provide options for altruistic ethical surrogacy to the
needy infertile Indian couples.
Q Source: http://timesofindia.indiatimes.com/india/After-
surrogacy-govt-to-regulate-IVF-sperm-
banks/articleshow/53881298.cms

1
WWW.INSIGHTSIAS.COM 0
1
INS INSIGHTS IAS PRELIMS TEST SERIES - 2017

92. It the largest National Park in Central India, and


known to inspire Rudyard Kipling for his famous novel
Jungle Book. It is the only place in India where its official
mascot, swamp deer, is found. This protected area is
located in?
a) Kanha
b) Keoladeo
c) Ranthambore
d) Bandipur
Solution: a)
Justification: Eliminate to reach the answer A. Ranthambore
and Keoladeo are in Rajasthan, and Bandipur is in Karnataka.
Learning: Kanha in Madhya Pradesh became first tiger
reserve in India to officially introduce a mascot named
Bhoorsingh the Barasingha (swamp deer), to create a better
connect with its visitors, especially the younger generation.

The mascot aims to present the hard ground swamp deer


(Barasingha) as the spirit of the reserve and spread
awareness to save it from possible extinction.
Swamp deer is the state animal of Madhya Pradesh. The
Kanha tiger reserve, spread over Mandla and Balaghat
districts is the only place in the world where the
Barasingha species exists.
The park has a significant population of Bengal tiger,
Indian leopards, sloth bear, barasingha and Indian wild
dog.

1
WWW.INSIGHTSIAS.COM 0
2
INS INSIGHTS IAS PRELIMS TEST SERIES - 2017

Q Source: National parks in news

93. With reference to colonial India, Lord Lytton is


credited with introducing
1. Local self-governance in panchayati raj institutions
2. Uniform salt tax throughout British India
3. Telegraph, press and missionaries in British India
Select the correct answer using the codes below.
a) 1 and 2 only
b) 2 and 3 only
c) 2 only
d) 1 and 3 only
Solution: c)
Justification: Eliminate.
Statement 1: This was famously introduced by Lord Ripon, who
is also known as the father of self-government in India. 1 is
incorrect.
Statement 3: This was famously introduced by Lord Dalhousie,
the harbinger of modern services like telegraph, posts and
railways in India.
Learning: Lytton abolished many import duties and
supported the Free Trade Policy which seriously affected Indian
economic interest. The system of decentralisation of finance
that had begun in the time of Lord Mayo was continued during
the time of Lord Lytton.
1
WWW.INSIGHTSIAS.COM 0
3
INS INSIGHTS IAS PRELIMS TEST SERIES - 2017

The provincial governments were empowered with some


control over the expenditure of all provincial matters like
land-revenue, excise, stamps, law and justice.
In 1878, the Statutory Civil Service was established
exclusively for Indians but this was abolished later.
Q Source: 12th TN Modern History Textbook

94. Which of the following were NOT introduced by the


Sakas and Kushanas in India?
a) Saka script, language and religion
b) Improved cavalry and horse riding
c) Hereditary dual rule system
d) Practice of wearing turbans and helmets
Solution: a)
Justification: The Sakas and Kushans added new ingredients
to Indian culture and enriched it immensely.
Option A: They settled in India for good and completely
identified themselves with its culture. Since they did not have
their script, Language or religion, they adopted these elements
of culture from India.
Option B: They introduced better cavalry and the use of the
riding horse on a large scale. They made common the use of
reins and saddles, which appear in the Buddhist sculptures of
the second and third centuries A D.
The Sakas and the Kushanas were excellent horsemen as well.

1
WWW.INSIGHTSIAS.COM 0
4
INS INSIGHTS IAS PRELIMS TEST SERIES - 2017

Option C: They introduced turban, tunic, trousers, and heavy


Long coat. Even now the Afghans and Panjabis wear turbans,
and the sherwani is a successor of the long coat. The Central
Asians also brought in cap, helmet and boots which were used
by warriors.
Option D: They strengthened the idea of the divine origin of
kingship. The Kushan kings were called sons of god.

They also introduced the satrap system of government.


The empire was divided into numerous satraps, and each
satrap was placed under the rule of a satrap. Some other
practices such as hereditary dual rule, two kings ruling in
the same kingdom at one and the same time, were
introduced.
Q Source: Ancient India: Old NCERT: RS Sharma

95. The String of Pearls theory, often seen in news,


concerns
a) Chinese military and commercial facilities in the Indian
Ocean
b) Oil pipelines being laid around the North-South
transport corridor
c) Valuable mineral and stone exploration licensing
regime in ocean beds
d) Network of diplomatic think tanks being established
across the world
Solution: a)

1
WWW.INSIGHTSIAS.COM 0
5
INS INSIGHTS IAS PRELIMS TEST SERIES - 2017

Learning: It refers to the network of Chinese military and


commercial facilities and relationships along its sea lines of
communication, which extend from the Chinese mainland to
Port Sudan.

The sea lines run through several major maritime choke


points such as the Strait of Mandeb, the Strait of Malacca,
the Strait of Hormuz, and the Lombok Strait as well as
other strategic maritime centres in Pakistan, Sri Lanka,
Bangladesh, the Maldives, and Somalia.
The Chinese government insists that China's burgeoning
naval strategy is entirely peaceful in nature and is only for
the protection of regional trade interests.

1
WWW.INSIGHTSIAS.COM 0
6
INS INSIGHTS IAS PRELIMS TEST SERIES - 2017

Q Source: http://www.thehindu.com/news/international/21-
injured-following-protests-against-Chinese-
project/article17006037.ece

96. Fractional Reserve banking system is utilized to


1. Avoid collapse of banks and banking systems
2. Control quantum of liquidity in the economy

1
WWW.INSIGHTSIAS.COM 0
7
INS INSIGHTS IAS PRELIMS TEST SERIES - 2017

3. Regulate the fiscal capacity of the government


4. Provide interest free loans to the most vulnerable
sections of society
Select the correct answer using the codes below.
a) 2, 3 and 4 only
b) 1 and 2 only
c) 3 and 4 only
d) 1, 2, 3 and 4
Solution: b)
Justification: Statement 1: The system essentially means that
Banks are required to keep a certain amount of the cash
depositors give them on hand available for withdrawal. That is,
if someone deposits Rs. 100, the bank can't lend out the entire
amount.

Many U.S. banks were forced to shut down during the


Great Depression because too many people attempted to
withdraw assets at the same time. This is to avoid such a
situation. CRR, SLR, CAR are some of the tools to control
it.
Statement 2: It follows logically from statement 1 that liquidity
is controlled.
Statement 3: It has no relation with regulating fiscal capacity of
government which is governments ability to earn revenues and
spend.
Statement 4: This is a part of priority sector lending and a part
of monetary policy, not fractional system.
Q Source: Chapter on Banking: Indian Economy: Ramesh
Singh

1
WWW.INSIGHTSIAS.COM 0
8
INS INSIGHTS IAS PRELIMS TEST SERIES - 2017

97. How does the National Wetland Conservation


Programme (NWCP) aim to achieve the conservation of
wetlands in India?
1. It prepares an inventory of Indian wetlands so that their
situation can be regularly monitored.
2. It finances the long-term comprehensive Management
Action Plans (MAPs) of states and Union territories
with regard to wetlands.
Which of the above is/are correct?
a) 1 only
b) 2 only
c) Both 1 and 2
d) None
Solution: c)
Justification: Statement 1: It also aims to lay down policy
guidelines for conservation and management of wetlands in the
country and undertake intensive conservation measures in
priority wetlands.
Statement 2: Financial assistance under NWCP is provided for
two components i.e. Management Action Plan (MAP) and
Research Projects. Under the Scheme, cent percent assistance is
provided for activities.
After identification of wetlands under the Scheme, the
State/UTs are to submit long-term comprehensive

1
WWW.INSIGHTSIAS.COM 0
9
INS INSIGHTS IAS PRELIMS TEST SERIES - 2017

Management Action Plans (MAPs) to be financed under the


programme.
Learning: All the states are also mandated to set up a State
Level Wetland Authority for wetland conservation, regulation
and management under the relevant State bye-laws.

This authority will be consisting of Chief Minister or


Minister in charge of environment and forests in state;
chief secretary of the state and secretary in the
departments of Forests, urban Development,
There is no specific law for wetland conservation,
management and use in India. The wetlands are covered
under Environment (Protection) Act, 1986.
Wetlands Rules Draft 2016 was notified last year.
Q Source: http://envfor.nic.in/division/national-wetland-
conservation-programme-nwcp

98. Brought to India in the 1850s, which of these was


considered as the Intellectual Charter of India?
a) Competitive Civil Services Examinations
b) Increased local representation in assemblies by direct
elections
c) Woods Dispatch on Educational Reforms
d) Proposal for administrative division of judiciary and
executive
Solution: c)

1
WWW.INSIGHTSIAS.COM 1
0
INS INSIGHTS IAS PRELIMS TEST SERIES - 2017

Learning: Dalhousie had evinced in the development of


education in India. The educational Despatch of Sir Charles
Wood (1854) was considered the Intellectual Charter of India.
It provided an outline for the comprehensive scheme of
education at primary, secondary and collegiate levels.
Dalhousie fully accepted the views of Charles Wood and took
steps to carry out the new scheme. Departments of Public
Instructions were organized. The Universities of Calcutta,
Bombay and Madras were founded in 1857.
Q Source: TN Textbook: 12th Standard: Modern History

99. The belt of ancient rocks of 2,000 million years from


Brazil coast matches with those from western Africa. Also,
the earliest marine deposits along the coastline of South
America and Africa are of the Jurassic age. These
observations validate which of these theories?
a) Sea Floor Spreading
b) Environmental Possibilism
c) Continental Drift Theory
d) Convectional Current Theory
Solution: c)
Justification & Learning:

1
WWW.INSIGHTSIAS.COM 1
1
INS INSIGHTS IAS PRELIMS TEST SERIES - 2017

According to Wegener, all the continents formed a single


continental mass, a mega ocean surrounded by the same.
The super continent was named PANGAEA, which meant
all earth. The mega-ocean was called PANTHALASSA,
meaning all water.
He argued that, around 200 million years ago, the super
continent, Pangaea, began to split. Subsequently, their
components broke up into various smaller continents that
exist today. This is the reason for the evidence as shown in
the figure above.
Q Source: Fundamental of Physical Geography: 11th NCERT

1
WWW.INSIGHTSIAS.COM 1
2
INS INSIGHTS IAS PRELIMS TEST SERIES - 2017

100. Which of these countries are sandwiched between


Saudi Arabia, Turkey and Iran without having any open
access to sea?
a) Oman, Yemen and Jordan
b) Syria, Iraq and Lebanon
c) Azerbaijan, Qatar and Kuwait
d) None of the above
Solution: d)
Justification: Syria and Lebanon open to Mediterranean Sea,
so B cant be the answer.
Jordan is landlocked, but there is no option containing this
alone.
Qatar and Kuwait have access to sea and arent sandwiched. So,
C is incorrect.
Same goes for Oman and Yemen. So, A is incorrect as well. See
the map below.

1
WWW.INSIGHTSIAS.COM 1
3
INS INSIGHTS IAS PRELIMS TEST SERIES - 2017

Q Source: Map based questions

1
WWW.INSIGHTSIAS.COM 1
4

Anda mungkin juga menyukai